Chapter 35: Spiritual Health

Chapter 35: Spiritual Health

MULTIPLE CHOICE

1. A co-worker asks the nurse to explain spirituality. What is the nurse’s best response?

a. It has a minor effect on health.
b. It is awareness of one’s inner self.
c. It is not as essential as physical needs.
d. It refers to fire or giving of life to a person.

ANS: B
Spirituality is often defined as an awareness of one’s inner self and a sense of connection to a higher being, to nature, or to some purpose greater than oneself. Spirituality is an important factor that helps individuals achieve the balance needed to maintain health and well-being and to cope with illness. Florence Nightingale believed that spirituality was a force that provided the energy needed to promote a healthy hospital environment and that caring for a person’s spiritual needs was just as essential as caring for his or her physical needs. The word spirituality comes from the Latin word spiritus, which refers to breath or wind. The spirit gives life to a  person.

2. The nurse is caring for a patient who professes to be an agnostic. Which information should the nurse consider when planning care for this patient belief system?

a. The patient is devoid of spirituality.
b. Is certain there is no such entity as God.
c. Believes there is no known ultimate reality.
d. Finds no meaning through relationships with others.

 

 

ANS: C
Some people do not believe in the existence of God (atheist), or they believe that there is no known ultimate reality (agnostic).
Nonetheless, spirituality is important regardless of a person’s religious beliefs. Agnostics discover meaning in what they do or how
they live because they find no ultimate meaning for the way things are. They believe that people bring meaning to what they do.
DIF: Understand (comprehension)
OBJ: Discuss the influence of spirituality on patients’ health practices.
TOP: Planning MSC: Management of Care
3. The nurse is caring for an Islamic patient who wants a snack. Offering sandwich is most appropriate?
a. Ham
b. Beef
c. Kosher
d. Bacon
ANS: B
Islam religion does allow beef. Islam does not allow pork or alcohol. Ham and bacon are pork. Kosher is allowed for Judaism.
DIF: Apply (application)
OBJ: Discuss nursing interventions designed to promote a patient’s spiritual health.
TOP: Implementation MSC: Psychosocial Integrity
4. A nurse is teaching a patient how to meditate. Which behavior presented by the patient indicates effective learning?
a. Lying on the floor
b. Breathing quickly
c. Focusing on an particular image
d. Engaging for 10 minutes every day
ANS: C
The steps of meditation include sitting in a comfortable position with the back straight; breathe slowly; and focus on a sound,
prayer, or image. Meditation should occur for 10 to 20 minutes twice a day.
DIF: Apply (application)
OBJ: Discuss nursing interventions designed to promote a patient’s spiritual health.
TOP: Teaching/Learning MSC: Psychosocial Integrity
5. The nurse is admitting a patient to the hospital. The patient is a very spiritual person but does not practice any specific religion.
How will the nurse interpret this finding?
a. This indicates a strong religious affiliation.
b. This statement is contradictory.
c. This statement is reasonable.
d. This indicates a lack of hope.
ANS: C
The patient’s statement is reasonable and is not contradictory. Many people tend to use the terms spirituality and religion
interchangeably. Although closely associated, these terms are not synonymous. Religious practices encompass spirituality, but
spirituality does not need to include religious practice. When a person has the attitude of something to live for and look forward to,
hope is present.
DIF: Apply (application)
OBJ: Compare and contrast the concepts of religion and spirituality.
TOP: Assessment MSC: Psychosocial Integrity
Copyright © 2021, Elsevier Inc. All rights reserved. 2
6. Which patient comment should the nurse identify as a demonstration of faith?
a. I go to church every Sunday.
b. I believe there is life after death.
c. I have something to look forward to each day.
d. I get a feeling of awe when looking at the sunset.
ANS: B
Faith allows people to have firm beliefs despite lack of physical evidence (life after death). Religion refers to the system of
organized beliefs and worship that a person practices to outwardly express spirituality (go to church). When a person has the
attitude of something to live for and look forward to, hope is present (look forward to each day). Self-transcendence is the belief
that there is a force outside of and greater than the person (awe when looking at a sunset).
DIF: Analyze (analysis)
OBJ: Describe the relationship among faith, hope, and spiritual well-being.
TOP: Assessment MSC: Psychosocial Integrity
7. A nurse is caring for a Roman Catholic patient. Which action will the nurse plan time for?
a. Practice the Five Pillars.
b. Practice Blessingway.
c. Avoiding meat on Fridays.
d. The purity rituals.
ANS: D
Fasting on Fridays is practiced by some Roman Catholics. Hindus practice prayer and purity rituals. Blessingway is a practice of
the Navajos that attempts to remove ill health by means of stories, songs, rituals, prayers, symbols, and sand paintings. Islams must
be able to practice the Five Pillars of Islam.
DIF: Apply (application)
OBJ: Discuss nursing interventions designed to promote a patient’s spiritual health.
TOP: Implementation MSC: Psychosocial Integrity
8. The nurse is caring for a patient with a chronic illness who is having conflicts with beliefs. Which health care team member will the
nurse ask to see this patient?
a. The clergy
b. A psychiatrist
c. A social worker
d. An occupational therapist
ANS: A
Other important resources to patients are spiritual advisors and members of the clergy. Spiritual care helps people identify meaning
and purpose in life, look beyond the present, and maintain personal relationships, as well as a relationship with a higher being or
life force. A psychiatrist is for emotional health. A social worker focuses on social, financial, and community resources. An
occupational therapist provides care with vocational issues and functioning within physical limitations.
DIF: Apply (application)
OBJ: Explain the importance of establishing caring relationships with patients to provide spiritual care. TOP: Implementation MSC:
Management of Care
9. The nurse caring for a terminally ill patient sits down and lightly touches the patient’s hand. Which technique is the nurse using?
a. “Doing for”
b. Establishing presence
c. Offering transcendence
d. Providing health promotion
ANS: B
Establishing presence by sitting with a patient to attentively listen to his or her feelings and situation, talking with the patient,
crying with the patient, and simply offering time are powerful spiritual care approaches. Benner explains that presence involves
“being with” a patient versus “doing for” a patient. Transcendence is the belief that a force outside of and greater than the person
exists beyond the material world. In settings where health promotion activities occur, patients often need information, counseling,
and guidance to make the necessary choices to remain healthy.
DIF: Apply (application)
OBJ: Identify approaches for establishing presence with patients.
TOP: Implementation MSC: Psychosocial Integrity
10. The nurse and the patient have the same religious affiliation. Which action will the nurse take to support the patient spiritually?
a. Must use a formal assessment tool to determine patient’s beliefs.
b. Assume that both have the same spiritual beliefs.
c. Do not impose personal values on the patient.
d. Skip the spiritual belief assessment.
ANS: C
It is important not to impose personal value systems on the patient. This is particularly true when the patient’s values and beliefs
are similar to those of the nurse because it then becomes very easy to make false assumptions. It is not a must to use a formal
assessment tool when assessing a patient’s beliefs. It is important to conduct the spiritual belief assessment; conducting an
assessment is therapeutic because it expresses a level of caring and support.
DIF: Apply (application)
OBJ: Explain the importance of establishing caring relationships with patients to provide spiritual care. TOP: Implementation MSC:
Psychosocial Integrity
Copyright © 2021, Elsevier Inc. All rights reserved. 3
11. A nurse makes a connection with the patient when providing spiritual care. Which type of connectedness did the nurse experience?
a. Intrapersonal
b. Interpersonal
c. Transpersonal
d. Multipersonal
ANS: B
Interpersonal means connected with others and the environment. Intrapersonal means connected within oneself. Transpersonal
means connected with God or an unseen higher power. There is no such term as multipersonal for connectedness.
DIF: Understand (comprehension)
OBJ: Explain the importance of establishing caring relationships with patients to provide spiritual care. TOP: Caring MSC: Psychosocial
Integrity
12. The patient is admitted with a diagnosis of chronic anxiety. Which action is most appropriate for the nurse to take when providing
holistic care?
a. Focusing on finding quick remedies for the anxiety
b. Realizing that the patient’s only goal is relief of the anxiety
c. Looking at how worry influences the patient’s ability to function
d. Helping the patient realize that there is little hope of relief from anxiety
ANS: C
Do not just look at the patient’s anxiety as a problem to solve with quick remedies, but rather look at how the anxiety influences the
patient’s ability to function and achieve goals established in life (not just anxiety relief). Mobilizing the patient’s hope is central to
a healing relationship.
DIF: Apply (application) OBJ: Assess a patient’s spirituality.
TOP: Caring MSC: Psychosocial Integrity
13. In caring for the patient’s spiritual needs, the nurse asks 20 questions to assess the patient’s relationship with God and a sense of
life purpose and satisfaction. Which method is the nurse using?
a. The spiritual well-being scale
b. The FICA assessment tool
c. Belief tool
d. Hope scale
ANS: A
The spiritual well-being scale (SWB) has 20 questions that assess a patient’s relationship with God and his or her sense of life
purpose and life satisfaction. The FICA assessment tool evaluates spirituality and is closely correlated to quality of life. This does
not describe belief or hope.
DIF: Understand (comprehension) OBJ: Assess a patient’s spirituality.
TOP: Assessment MSC: Psychosocial Integrity
14. A male patient in stable condition is in the intensive care unit (ICU) and is asking to see his spouse and two daughters. What should
the nurse be prepared to plan for?
a. 5 to 10 minutes for the family visit
b. Complying with the patient’s request
c. Arranging for the two daughters to visit, then the wife
d. Asking the family to decide which two persons will visit
ANS: B
Use of support systems is important in any health care setting. Allowing the family to visit is appropriate since the patient is in
stable condition. When patients depend on family and friends for support, encourage them to visit the patient. As long as no
interference with active patient care is involved, there is no reason to limit visitation. Limiting the visit is not necessary since the
patient is stable. Breaking the family apart is not needed; the patient is stable and can see all three at once.
DIF: Apply (application)
OBJ: Discuss nursing interventions designed to promote a patient’s spiritual health.
TOP: Implementation MSC: Psychosocial Integrity
15. The nurse is caring for a patient who has been diagnosed with a terminal illness. The patient states, “I just don’t feel like going to
work. I have no energy, and I can’t eat or sleep.” The patient shows no interest in taking part in the care by saying, “What’s the
use?” Which response by the nurse is best?
a. It sounds like you have lost hope.
b. It sounds like you have lost energy.
c. It sounds like you have lost your appetite.
d. It sounds like you have lost the ability to sleep.
ANS: A
All of the patient’s description are describing a loss of hope. While losses of energy, appetite, and sleep are indicated, they only
address a part of patient’s problems. A loss of hope encompasses the holistic view of the patient.
DIF: Apply (application)
OBJ: Explain the importance of establishing caring relationships with patients to provide spiritual care. TOP: Caring MSC: Psychosocial
Integrity
Copyright © 2021, Elsevier Inc. All rights reserved. 4
16. The patient is having a difficult time dealing with an AIDS diagnosis. The patient states, “It’s not fair. I’m totally isolated from
God and my family because of this. Even my father hates me for this. He won’t even speak to me.” What should the nurse do?
a. Tell the patient to move on and focus on getting better.
b. Use therapeutic communication to establish trust and caring.
c. Assure the patient that the father will accept this situation soon.
d. Point out that the patient has no control and that he or she must face the
consequences.
ANS: B
Application of therapeutic communication principles and caring helps you establish therapeutic trust with patients. The nurse
should not offer false hope (father will accept the situation soon). The nurse should help the patient maintain feelings of control, not
no control. The nurse should encourage renewing relationships if possible and establishing connections with self, significant others,
and God.
DIF: Apply (application)
OBJ: Discuss nursing interventions designed to promote a patient’s spiritual health.
TOP: Implementation MSC: Psychosocial Integrity
17. The nurse is caring for a group of patients. Which patient will the nurse see first?
a. A patient saying that God has left and there is no reason for living
b. A patient refusing treatment on the Sabbath
c. A patient having a folk healer in the room
d. A patient praying to Allah
ANS: A
A patient saying that God has left and there is no reason for living must be seen first for safety reasons. It must be determined by
the nurse if the patient is planning suicide or is just angry and frustrated. A patient refusing treatment on the Sabbath is within that
patient’s right and doesn’t need to be seen first. A patient with a folk healer is within the patient’s right and does not need to be
seen first. A patient praying to Allah is within the patient’s right and does not need to be seen first.
DIF: Analyze (analysis) OBJ: Assess a patient’s spirituality.
TOP: Assessment MSC: Management of Care
18. A nurse is providing spiritual care to patients. Which action is essential for the nurse to take?
a. Knowing one’s own personal beliefs
b. Learning about other religions
c. Visit churches, temples, mosques, or synagogues
d. Travel to other areas that do not have the same beliefs
ANS: A
Because each person has a unique spirituality, you need to know your own beliefs, so you are able to care for each patient without
bias. While learning about religions, visiting other religious areas of worship, and traveling to areas that do not have the same
beliefs are beneficial, they are not essential.
DIF: Understand (comprehension)
OBJ: Explain the importance of establishing caring relationships with patients to provide spiritual care. TOP: Implementation MSC:
Psychosocial Integrity
MULTIPLE RESPONSE
1. A nurse is evaluating a patient’s spiritual care. Which areas will the nurse include in the evaluation process? (Select all that apply.)
a. Review the patient’s view of the purpose in life.
b. Ask whether the patient’s expectations were met.
c. Discuss with family and friends the patient’s connectedness.
d. Review the patient’s self-perception regarding spiritual health.
e. Impress on the patient that spiritual health is permanent once obtained.
ANS: A, B, C, D
In evaluating care include a review of the patient’s self-perception regarding spiritual health, the patient’s view of his or her
purpose in life, discussion with the family and friends about connectedness, and determining whether the patient’s expectations
were met. Attainment of spiritual health is a lifelong goal; it is not permanent once obtained.
DIF: Apply (application)
OBJ: Evaluate patient outcomes related to spiritual health. TOP: Evaluation
MSC: Management of Care
2. Spiritual distress has been identified in a patient who has been diagnosed with a chronic illness. Which interventions will the nurse
add to the care plan? (Select all that apply.)
a. Being supportive of a patient’s wish to pray
b. Encouraging time with the support group
c. Arranging for a clergy member to visit the patient
d. Developing activities to heal body, mind, and spirit
e. Teaching relaxation, guided imagery, and meditation
ANS: A, D, E
Interventions that are appropriate for spiritual distress include: (1) helping the patient develop/identify activities to heal body, mind,
and spirit; (2) supporting the patient’s desire to pray; (3) encourage attendance with an appropriate support group; and (4) teaching
relaxation, guided imagery, and medication. Clergy consults should be made only with the patient’s permission.
DIF: Apply (application)
OBJ: Discuss nursing interventions designed to promote a patient’s spiritual health.
TOP: Planning MSC: Management of Care
Copyright © 2021, Elsevier Inc. All rights reserved. 5
MATCHING
A nurse is providing spiritual care to a group of patients. Match the group to their belief.
a. Avoid caffeine.
b. Says prayers five times a day.
c. Shellfish is not eaten.
d. Often follow a vegetarian diet.
e. Generally fast on religious Holy days.
1. Hinduism
2. Buddhism
3. Islam
4. Judaism
5. Mormonism
1. ANS: D DIF: Understand (comprehension)
OBJ: Discuss the influence of spirituality on patients’ health practices.
TOP: Caring MSC: Management of Care
2. ANS: E DIF: Understand (comprehension)
OBJ: Discuss the influence of spirituality on patients’ health practices.
TOP: Caring MSC: Management of Care
3. ANS: B DIF: Understand (comprehension)
OBJ: Discuss the influence of spirituality on patients’ health practices.
TOP: Caring MSC: Management of Care
4. ANS: C DIF: Understand (comprehension)
OBJ: Discuss the influence of spirituality on patients’ health practices.
TOP: Caring MSC: Management of Care
5. ANS: A DIF: Understand (comprehension)
OBJ: Discuss the influence of spirituality on patients’ health practices.
TOP: Caring MSC: Management of Care

Chapter 33: Self-Concept

Chapter 33: Self-Concept

MULTIPLE CHOICE

1. When caring for an older adult patient, which technique will the nurse use to enhance an older-adult patient’s self-concept?

a. Discussing current weather
b. Encouraging patients to sing
c. Reviewing old photos with patients
d. Allowing patients extra computer time

 

ANS: C
Nurses can improve self-concept by reviewing old photographs when working with older-adult patients. This form of life review is helpful to older adults in remembering positive life events and people. Discussing weather does not involve personal reflection. Singing does not improve self-concept. Giving patients extra computer time is not applicable to improving self-concept but may help with learning.

 

2. While gathering an adolescent’s health history, the nurse recognizes that the patient began to act out behaviorally and engaged in risky behavior when the patient’s parents divorced. Which information will the nurse gather to help in determining situational low self-esteem?

a. How long the parents were married?
b. How the patient views behaviors?
c. Why the parents are divorcing?
d. Why the patient is acting out of control?

ANS: B
A nurse can identify situational life stressors that can impact a person’s self-concept. By asking about a patient’s thoughts and
feelings, the nurse will be able to use communication skills in a therapeutic manner. This will facilitate the patient’s insight into
behaviors and will enable the nurse to make referrals or provide needed health teaching. The length of time married and the reason
for the parents’ divorce do not explain the patient’s behaviors. Why the patient is acting out of control is not as important as how
the patient views actions when out of control.
DIF: Analyze (analysis)
OBJ: Apply the nursing process to promote a patient’s self-concept.
TOP: Assessment MSC: Psychosocial Integrity
3. A nurse is assessing a group of adolescents. Which person is most likely to have the highest self-esteem?
a. Latino adolescent female who has strong ethnic pride.
b. Caucasian boy who lives below federal poverty level.
c. African-American adolescent male who has severe acne.
d. Adolescent who was suspended twice from high school.
ANS: A
When cultural identity is central to self-concept and is positive, cultural pride and self-esteem tend to be strong. Environmental
stressors such as low-income, body image stressors such as acne, and role performance failure often influence self-esteem
negatively.
DIF: Understand (comprehension)
OBJ: Examine cultural considerations that affect self-concept. TOP: Assessment
MSC: Psychosocial Integrity
4. A nurse is caring for a patient who is dealing with the developmental task known as initiative versus guilt. The nurse is providing
care to which patient?
a. A 3-week-old neonate
b. A 5-year-old kindergarten student
c. An 11-year-old student
d. A 15-year-old high school student
ANS: B
The initiative versus guilt developmental stage occurs between the ages of 3 and 6 years. The patient is a 5-year-old kindergarten
student. If a child shows initiative, the outcome of this developmental task is to develop purpose. A neonate developmental task is
to develop trust. An 11-year-old student is into new skill mastery (industry), and a 15-year-old student is struggling with identity
versus role confusion.
DIF: Understand (comprehension)
OBJ: Describe the components of self-concept as related to psychosocial and cognitive developmental stages. TOP:
Implementation
MSC: Health Promotion and Maintenance
Copyright © 2021, Elsevier Inc. All rights reserved. 2
5. A verbally abusive partner has told a significant other many negative comments over the years. In the crisis center, the nurse would
anticipate that the patient may have which self-concept deficits?
a. Body image
b. Role confusion
c. Rigidity
d. Yearning
ANS: A
Over the years of marriage, the significant other incorporates this devaluation into his or her own self-concept, negatively affecting
body image. The way others view a person’s body and the feedback offered are also influential on body image and self-concept.
Role confusion is part of a developmental task (identity versus role confusion) for adolescents. Rigidity and yearning are not
components of self-concept.
DIF: Understand (comprehension)
OBJ: Discuss factors that influence the components of self-concept.
TOP: Assessment MSC: Psychosocial Integrity
6. Two 50-year-old men are discussing their Saturday activities. The first man describes how he tutors children as a volunteer at a
community center. The other man says that he would never work with children and that he prefers to work out at the gym to meet
young women to date. Which developmental stage is the second man exhibiting?
a. Mistrust
b. Inferiority
c. Generativity
d. Self-absorption
ANS: D
In the generativity versus self-absorption developmental task, a self-absorbed person is concerned about own personal wants and
desires in a self-centered manner. Generativity is the first man’s developmental stage. Trust versus mistrust occurs in the first year
of life. Industry versus inferiority commonly occurs in school children. Identity versus role confusion commonly occurs at the start
of adolescence into young adulthood.
DIF: Apply (application)
OBJ: Describe the components of self-concept as related to psychosocial and cognitive developmental stages. TOP:
Assessment
MSC: Health Promotion and Maintenance
7. An adult son is adjusting to the idea of his chronically ill parents moving into his family home. The community health nurse is
assessing the adult son for potential stressors secondary to the new family living arrangement. Which stressor will the nurse assess
for in this adult son?
a. Role confusion
b. Role ambiguity
c. Role performance
d. Role overload
ANS: D
Role overload involves having more roles or responsibilities within a role than are manageable. Role overload is common in
individuals who unsuccessfully attempt to meet the demands of work and family while trying to find some personal time. Role
confusion is an aspect of the developmental task of adolescence and young adulthood (identity versus role confusion). Role
ambiguity involves unclear role expectations. Role performance is the way in which individuals perceive their ability to carry out
significant roles; it is not a stressor unless it is judged ineffective. There is no data in the question to indicate this.
DIF: Apply (application)
OBJ: Identify stressors that affect self-concept and self-esteem. TOP: Assessment
MSC: Psychosocial Integrity
8. A nurse grimaces while changing a patient’s colostomy bag. Which effect will the nurse’s behavior most likely have on the
patient?
a. Assist recovery by using honest communication.
b. Motivate the patient to increase physical activity.
c. Promote development of a negative body image.
d. Develop a kind nickname for the colostomy bag.
ANS: C
Negative nonverbal reactions by a nurse to a patient’s scar or surgical alterations contribute to the patient’s developing a negative
body image. Nurses who have shocked or disgusted facial expressions contribute to patients’ developing a negative body image.
Expressions of distaste by the nurse will not facilitate recovery or ongoing communication, encourage physical activity, or promote
acceptance of the colostomy bag by adopting a positive nickname.
DIF: Understand (comprehension)
OBJ: Explore ways in which a nurse’s self-concept and nursing actions affect a patient’s self-concept and self-esteem. TOP: Evaluation
MSC: Psychosocial Integrity
Copyright © 2021, Elsevier Inc. All rights reserved. 3
9. A male patient states, “I’m such a loser. I only had that job for a month.” Which outcome criteria will the nurse add to the patient’s
care plan?
a. The patient will verbalize two life areas in which he functions well.
b. The patient will find new employment before the next clinic visit.
c. The patient will confront a former boss about previous work problems.
d. The patient will identify two reasons why he is considered a bad employee.
ANS: A
Verbalizing two life areas in which a person functions well is an individualized measurable outcome that is realistic. Confronting a
former boss could have physical and emotional repercussions for the patient. If the patient is voicing that he has problems obtaining
employment, then putting extra pressure to obtain employment would be detrimental to the patient and does not reflect a supportive
and caring nursing outcome. Focusing on the negative of why the patient is considered a bad employee is not as beneficial as
focusing on strengths.
DIF: Analyze (analysis)
OBJ: Apply the nursing process to promote a patient’s self-concept.
TOP: Planning MSC: Psychosocial Integrity
10. A nurse is teaching a patient about self-concept. Which information from the patient indicates the nurse needs to follow up about
components of self-concept?
a. One component is identity.
b. One component is coping.
c. One component is body image.
d. One component is role performance.
ANS: B
The nurse will need to follow up for the information that a component of self-concept is coping; this is a misconception and must
be corrected. The components of self-concept are identity, body image, and role performance. While self-concept may affect
coping, coping is not a component of self-concept.
DIF: Understand (comprehension)
OBJ: Describe the components of self-concept as related to psychosocial and cognitive developmental stages. TOP:
Teaching/Learning
MSC: Management of Care
11. Which individual is most likely to need the nurse’s assistance in coping with identity confusion?
a. A 49-year-old male with stable employment
b. A 35-year-old recently divorced mother of twins
c. A 22-year-old in the third year of college
d. A 50-year-old self-employed woman
ANS: B
Identity confusion results when people do not maintain a clear, consistent, and continuous consciousness of personal identity. A
newly divorced woman would be trying to adapt to a new lifestyle of being single while handling parenting of twins as a single
parent. This situation could lead to identity confusion. A college sophomore would have had at least 2 years to adjust to the new
life setting, and a self-employed woman would likely be content with creating her own employment opportunity. There is no
indication that the middle-aged man with stable employment should have identity confusion.
DIF: Apply (application)
OBJ: Discuss evidence-based practice applicable for identity confusion, disturbed body image, low self-esteem, and role conflict.
TOP: Planning MSC: Psychosocial Integrity
12. A nurse is assessing a patient for possible altered self-concept. Which assessment finding is consistent with altered self-concept?
a. Appropriately dressed in clean clothes
b. Hesitant to express opinions
c. Independent attitude
d. Holds eye contact
ANS: B
Hesitant to express views or opinions is a behavior suggestive of altered self-concept. Holds eye contact, independent attitude, and
appropriate appearance are all signs of normal self-concept.
DIF: Apply (application)
OBJ: Apply the nursing process to promote a patient’s self-concept.
TOP: Assessment MSC: Psychosocial Integrity
Copyright © 2021, Elsevier Inc. All rights reserved. 4
13. A nurse is caring for a postoperative mastectomy patient. Which action is a priority for increasing self-awareness?
a. Solving problems for the patient before developing insight
b. Using communication skills to clarify family and patient expectations
c. Telling the patient that it will be fine because many others have survived
d. Rotating nursing personnel in the patient’s care, so the patient can talk to many
people
ANS: B
Increase the patient’s self-awareness by allowing him or her to openly explore thoughts and feelings. A priority nursing
intervention is the expert use of therapeutic communication skills to clarify the expectations of a patient and family. Interventions
designed to help a patient reach the goal of adapting to changes in self-concept or attaining a positive self-concept are based on the
premise that the patient first develops insight and self-awareness concerning problems and stressors and then acts to solve the
problems and cope with the stressors. Reassurance that a person will do fine dismisses any potential concerns the patient may have.
Rotating nursing personnel does not allow time for the patient to build rapport with any one nurse.
DIF: Apply (application)
OBJ: Apply the nursing process to promote a patient’s self-concept.
TOP: Implementation MSC: Psychosocial Integrity
14. The nurse in an addictions clinic is working with a patient on priority setting before the patient’s discharge from residential
treatment. Which goal is a priority at this time?
a. Identifying local self-help groups before being discharged from the program
b. Stating a plan to never be tempted by illicit substances after discharge
c. Staying away from all triggers that cause substance abuse
d. Recognizing personal areas of weakness to grow stronger
ANS: A
Look for strengths in both the individual and the family and provide resources and education to turn limitations into strengths, such
as local self-help groups. It is not realistic to avoid ALL triggers that can result in addictive behaviors. It is unrealistic to believe
that the patient will never be tempted because temptation can arise from multiple sources. On the other hand, an appropriate
priority would be to recognize that triggers will arise and that the patient should learn how to handle being confronted in the
post-discharge setting. Having a person talk about weaknesses without recognizing a person’s strengths could be a trigger to return
to an addictive lifestyle, so this would not be the most appropriate priority.
DIF: Analyze (analysis)
OBJ: Apply the nursing process to promote a patient’s self-concept.
TOP: Planning MSC: Psychosocial Integrity
15. A nurse is teaching a patient about self-concept. Which information from the patient indicates a correct understanding of the
teaching?
a. Self-concept is how a person feels about others.
b. Self-concept is how a person thinks about others.
c. Self-concept is how a person feels about oneself.
d. Self-concept is how a person thinks about oneself.
ANS: D
Self-concept, or how a person thinks about oneself, directly affects self-esteem, or how one feels about oneself. While others may
influence self-concept, self-concept is not how one feels or thinks about others.
DIF: Understand (comprehension)
OBJ: Discuss factors that influence the components of self-concept.
TOP: Teaching/Learning MSC: Psychosocial Integrity
16. A nurse is evaluating a patient’s self-concept. Which key indicator will the nurse use?
a. Drug abuse history
b. Nonverbal behavior
c. Personal journal entries
d. Posts on social media
ANS: B
Key indicators of a patient’s self-concept are nonverbal behaviors. A history of drug abuse does not necessarily indicate current
self-concept, and people who do not have a drug abuse history may have a low self-concept. It would be an invasion of privacy and
trust for a nurse to read a patient’s personal journal or social media posts.
DIF: Apply (application)
OBJ: Apply the nursing process to promote a patient’s self-concept.
TOP: Evaluation MSC: Psychosocial Integrity
17. A nurse is assessing a patient’s self-concept. Which area should the nurse assess first?
a. Role performance
b. Vital signs
c. Anxiety
d. Morals
ANS: A
In assessing self-concept and self-esteem, first focus on each component of self-concept (identity, body image, and role
performance). Self-concept is a psychological/emotional issue, not a physical issue for vital signs. Anxiety may be a stressors or a
sign of low self-concept. Self-concept is not a moral issue.
DIF: Apply (application)
OBJ: Apply the nursing process to promote a patient’s self-concept.
TOP: Assessment MSC: Psychosocial Integrity
Copyright © 2021, Elsevier Inc. All rights reserved. 5
18. A 9 year old is proudly telling the nurse about mastering the yellow belt in a martial arts class. Which developmental stage is the
child exhibiting?
a. Initiative versus guilt
b. Industry versus inferiority
c. Identity versus role confusion
d. Autonomy versus shame and doubt
ANS: B
Industry versus inferiority occurs between the ages of 6 and 12 years. It is during this developmental task that a person gains
self-esteem through new skill mastery. Initiative versus guilt is for 3 to 6 years, focusing on increasing language skills with
identification of feelings. Identity versus role confusion is 12 to 20 years, focusing on finding a sense of self. Autonomy versus
shame and doubt is 1 to 3 years, focusing on becoming more independent.
DIF: Apply (application)
OBJ: Describe the components of self-concept as related to psychosocial and cognitive developmental stages. TOP:
Assessment
MSC: Health Promotion and Maintenance
19. A nurse is developing a drinking prevention presentation for adolescents. Which areas should the nurse include in the teaching
session?
a. Stressful life events and scholarships
b. Very high self-esteem and work failure
c. Health problems and avoidance of conflict
d. Stress management and improving self-esteem
ANS: D
Drinking prevention efforts should include stress management and improving self-esteem. High self-esteem decreases risk of
drinking. Stressful life events when balanced with positive issues, such as receipt of a scholarship, are less likely to induce
drinking. Conflict resolution can strengthen adolescent coping strategies to decrease drinking.
DIF: Apply (application)
OBJ: Discuss evidence-based practice applicable for identity confusion, disturbed body image, low self-esteem, and role conflict.
TOP: Teaching/Learning
MSC: Health Promotion and Maintenance
20. A nurse is completing a history on a patient with role conflict. Which finding is consistent with role conflict?
a. A patient is unsure about job expectations in a fast-paced company.
b. A patient has to travel for work and misses children’s birthdays.
c. A patient feels less of a man after a leg amputation.
d. A patient loses a job from the company’s downsizing.
ANS: B
Role conflict results when a person has to simultaneously assume two or more roles that are inconsistent, contradictory, or mutually
exclusive—for example, when a patient has to travel for work and misses children’s birthdays. Role ambiguity is also common in
employment situations. In complex, rapidly changing, or highly specialized organizations, employees often become unsure about
job expectations. Feeling less of a man after a leg amputation is a body image and self-concept/self-esteem problem. Losing a job
can lead to low self-esteem or loss of job identity.
DIF: Analyze (analysis)
OBJ: Discuss evidence-based practice applicable for identity confusion, disturbed body image, low self-esteem, and role conflict.
TOP: Assessment MSC: Psychosocial Integrity
MULTIPLE RESPONSE
1. A nurse is assessing a patient’s self-concept. Which areas will the nurse include? (Select all that apply.)
a. Identity
b. Body image
c. Coping behaviors
d. Significant others’ support
e. Availability of insurance
ANS: A, B, C, D
Assessment of self-concept includes identity, body image, coping behaviors, and significant others’ support. Availability of
insurance is not a component of self-concept.
DIF: Understand (comprehension)
OBJ: Apply the nursing process to promote a patient’s self-concept.
TOP: Assessment MSC: Psychosocial Integrity
MATCHING
A nurse is assessing a group of patients. Match the assessment finding to the area the nurse is assessing.

a. I am ugly with all these burn scars.
b. I am one with the universe.
c. I am good for nothing.
d. I am a good mother.
1. Identity confusion
2. Disturbed body image
3. Role performance
4. Low self-esteem

 

 

1. ANS: B
2. ANS: A
3. ANS: D
4. ANS: C

Chapter 49: Sensory Alterations

Chapter 49: Sensory Alterations

MULTIPLE CHOICE

1. A nurse is administering a vaccine to a child who is visually impaired. After the needle enters the arm, the child says, “Ow, that was sharp!” What conclusion about the child will the nurse come to when interpreting the child’s response?

a. Sensations are intact.
b. Reception is intact.
c. Perception is intact.
d. Reaction is intact.

ANS: C
When a person becomes conscious of a stimulus and receives the information, perception takes place. Perception includes the integration and interpretation of stimuli based on the person’s experiences. Sensation is a general term that refers to awareness of sensory stimuli through the body’s sense mechanisms. Reception begins with stimulation of a nerve cell called a receptor, which is usually for only one type of stimulus such as light, touch, taste, or sound. The reaction is how a person responds to a perceived stimulus.

2. A nurse is describing the transmission of sound to a patient. In which order will the nurse list the pathway of sound, beginning with the first structure?

1. Eardrum
2. Perilymph
3. Oval window
4. Bony ossicles
5. Eighth cranial nerve
a. 1, 5, 2, 4, 3
b. 1, 3, 4, 2, 5
c. 1, 2, 4, 5, 3
d. 1, 4, 3, 2, 5

 

 

ANS: D
The vibration of the eardrum transmits through the bony ossicles. Vibrations at the oval window transmit in perilymph within the inner ear to stimulate hair cells that send impulses along the eighth cranial nerve to the brain.

 

3. A nurse is caring for a patient diagnosed with presbycusis. Which assessment does finding indicate an adaptation to the sensory deficit?

a. The patient frequently cleans out eyes with saline washes.
b. The patient applies different spices during mealtime to food.
c. The patient turns one ear toward the nurse during the conversation.
d. The patient isolates themself from social situations with groups of people.

 

ANS: C
Presbycusis is impaired hearing due to the aging process. Adaptation for a sensory deficit indicates that the patient alters behavior to accommodate for the sensory deficit, such as turning the unaffected ear toward the speaker. Cleaning the eye and applying spices to food would not have an effect for a patient with presbycusis. Avoiding others because of a sensory deficit is maladaptive.

 

4. The nurse will be most concerned about the risk of malnutrition for a patient with which sensory deficit?

a. Xerostomia
b. Dysequilibrium
c. Diabetic retinopathy
d. Peripheral neuropathy

 

ANS: A
Xerostomia is a decrease in the production of saliva; this decreases the ability and desire to eat and can lead to nutritional problems. The other options do not address taste- or nutrition-related concerns. Dysequilibrium is balance. Diabetic retinopathy affects vision. Peripheral neuropathy includes numbness and tingling of the affected areas and stumbling gait.

 

5. A nurse is caring for an older adult. Which sensory change will the nurse identify as normal during the assessment?

a. Impaired night vision
b. Difficulty hearing low pitch
c. Heightened sense of smell
d. Increased taste discrimination

 

 

ANS: A
Night vision becomes impaired as physiological changes in the aging eye occur. Older adults lose the ability to distinguish high-pitched noises and consonants. Senses of smell and taste are also decreased with aging.

 

6. A nurse is caring for an older adult patient who was in a motor vehicle accident because the patient thought the stoplight was green. The patient asks the nurse “Should I stop driving?” Which response by the nurse is most therapeutic?

a. “Yes, you should stop driving. As you age, your cognitive function declines, and becoming confused puts everyone else on the road at risk.”
b. “Yes, you should ask family members to drive you around from now on. Your
reflex skills have declined so much you can’t avoid an accident.”
c. “No, as you age, you lose the ability to see colors. You need to think about stoplights in a new way. If the top is lit up, it means stop, and if the bottom is lit
up, it means to go.”
d. “No, instead you should see your ophthalmologist and get some glasses to help you see better.”

 

 

ANS: C
Part of the normal aging process is the reduced ability to see colors. The nurse should teach the patient new ways to adapt to this deficit. This patient’s accident was not due to impaired cognitive function or reflexes. Glasses will not assist the patient in color discrimination.

 

7. A patient who recently had a stroke is going to be discharged at the end of the week. The nurse notices that the patient is having difficulty with communication and becomes tearful at times. Which intervention will the nurse include in the patient’s plan of care?

a. Teach the patient about special assistive devices.
b. Make the patient talk as much as possible.
c. Obtain an order for antidepressant medications.
d. Place a consult for a home health nurse.

 

 

ANS: A
Because a stroke often causes partial or complete paralysis of one side of a patient’s body, the patient needs special assistive devices. The nurse should include interventions that help the patient adapt to this deficit while maintaining independence. Teaching the patient to use assistive devices allows the patient to care for him- or herself. Making the patient talk can be inappropriate and demeaning. A home health nurse is not necessary as long as the patient is able to care for him- or herself. Instead of placing the patient on antidepressants, assist the patient in attempting to adapt behavior to the sensory deficit.

 

8. A patient has both hearing and visual sensory impairments. Which psychological nursing diagnosis will the nurse add to the care plan?

a. Risk for falls
b. Self-care deficit
c. Impaired socialization
d. Impaired physical mobility

 

ANS: C
In focusing on the psychological aspect of care, the nurse is most concerned about social isolation for a patient who may have difficulty communicating owing to visual and hearing impairment. Self-care deficit, impaired physical mobility, and fall risk are physiological risks for the patient.

 

9. During an assessment, the nurse finds the patient experiences vertigo. Which sensory deficit will the nurse assess further?

a. Neurological deficit
b. Visual deficit
c. Hearing deficit
d. Balance deficit

 

 

ANS: D
Vertigo is a result of vestibular dysfunction and often is precipitated by a change in head position. Neurological deficits include peripheral neuropathy and stroke. Visual deficits include presbyopia, cataracts, glaucoma, and macular degeneration. Hearing deficits include presbycusis and cerumen accumulation.

 

10. A home health nurse is assembling a puzzle with an older adult patient and notices that the patient is having difficulty connecting two puzzle pieces. Which aspect of sensory deprivation will the nurse document as being most affected?

a. Perceptual
b. Cognitive
c. Affective
d. Social

 

ANS: A
Alterations in spatial orientation and in visual/motor coordination are signs of perceptual dysfunction. Cognitive function is the ability to think and the capacity to learn; the patient is not disoriented or unable to learn. Affective problems include boredom and restlessness; the patient is participating in an activity. The patient is interacting with the home health nurse, so socialization is not a problem.

 

11. Which assessment question should the nurse ask to best understand how visual alterations are affecting the patient’s self-care ability?

a. “Have you stopped reading books or switched to books on audiotape?”
b. “What do you do to protect yourself from injury at work?”
c. “Are you able to prepare a meal or write a check?”
d. “How does your vision impairment make you feel?”

 

 

ANS: C
To best understand how vision is affecting self-care ability, the nurse wants to target questions to encompass what self-care tasks the patient has difficulty doing, such as preparing meals and writing checks. Switching to books on audiotape gives the nurse an idea of the severity of the deficit but not its impact on activities of daily living. Assessing whether the patient is taking measures for protection is important, but this does not address self-care activities. Emotional assessment of a patient is also important but does
not properly address the goal of determining the effect of visual alterations on self-care ability.

 

12. A nurse is assessing the cognitive functioning of a patient. Which action will the nurse take?

a. Administer a Mini-Mental State Examination (MMSE).
b. Ask the patient to state name, location, and what month it is.
c. Ask the patient’s family if the patient is behaving normally.
d. Administer the hearing handicap inventory for the elderly (HHIE-S).

 

 

ANS: A
The MMSE is a formal diagnostic tool that is used to assess a patient’s level of cognitive functioning. The Mini-Mental State Examination (MMSE) is a tool you can use to measure disorientation, change in problem-solving abilities, and altered conceptualization and abstract thinking. Asking the patient orientation questions evaluates only the patient’s orientation to self and surroundings, not abstract reasoning or critical thinking ability. Family members are not the most reliable source of information about the patient, although information received from the family should be considered. The HHIE-S is a 5-minute, 10-item
the questionnaire that assesses how the individual perceives the social and emotional effects of hearing loss. The higher the HHIE-S score, the greater the handicapping effect of hearing impairment.

 

13. Which patient is demonstrating a refractive error sensory problem?
a. A patient who frequently reports the incorrect time from the clock across the room.
b. A patient who is having difficulty remembering how to perform familiar tasks.
c. A patient who turns the television up as loud as possible.
d. A patient who has trouble saying words.

 

 

 

ANS: A
The most common visual problem is a refractive error such as nearsightedness. Difficulty remembering how to perform familiar tasks indicates the need to further assess mental and cognitive status. Turning the television up louder indicates the need for a hearing assessment. For a patient having trouble saying words, a picture board/chart may be used.

 

 

14. A nurse is caring for a patient who is undergoing chemotherapy for cancer. The patient is becoming malnourished because nothing tastes good. Which recommendation by the nurse will be most appropriate for this patient?

a. “Rinse your mouth several times a day to hydrate your taste buds.”
b. “Avoid adding spices or lemon juice to food to prevent nausea.”
c. “Blend foods together in interesting flavor combinations.”
d. “Eat soft foods that are easy to chew and swallow.”

 

 

ANS: A
Good oral hygiene keeps the taste buds well hydrated. Having an unpleasant taste in the mouth discourages the patient from eating. Well-seasoned, differently textured food eaten separately heightens taste perception. Avoid blending foods together because this makes it difficult to identify tastes. Texturized, spicy, and aromatic foods stimulate and make eating more enjoyable. Flavored vinegar or lemon juice adds tartness to food.

 

 

15. The nurse is creating a plan of care for a patient diagnosed with glaucoma. Which nursing diagnosis will the nurse include in the care plan to address a safety complication of the sensory deficit?

a. Body image disturbance
b. Impaired socialization
c. Risk for falls
d. Fear

 

 

ANS: C
A visual disturbance poses a great risk for injury due to falling from impaired depth perception and inability to see obstacles. Body image disturbance, social isolation, and fear are all valid nursing diagnoses that apply to a patient with a vision deficit; however, they do not address the greatest risk for injury.

 

16. The nurse is caring for a patient who is having difficulty understanding the written and spoken word. Which type of aphasia will the nurse report to the oncoming shift?

a. Expressive
b. Receptive
c. Global
d. Motor

 

 

ANS: B
Sensory or receptive aphasia is the inability to understand written or spoken language. A patient is able to express words but is unable to understand the questions or comments of others. Expressive aphasia, a motor type of aphasia, is the inability to name common objects or express simple ideas in words or writing. Global aphasia is the inability to understand language or communicate
orally.

 

17. The nurse is caring for a patient with conductive hearing loss resulting from prolonged cerumen impaction. Which intervention by the nurse is most important in establishing effective communication with the patient?

a. Speaking with hands, face, and expressions
b. Using a loud voice, enunciating every syllable
c. Having a direct conversation with the patient in the affected ear
d. Repeating the phrase again, if the patient does not understand what the nurse said

 

 

ANS: A
Use visible expressions. Speak with your hands, your face, and your eyes. Do not shout. Speaking in loud tones can distort a patient’s ability to hear; the nurse should speak in normal low tones. If the patient does not understand the first time, try rephrasing instead of repeating the message. The nurse can direct the conversation toward the patient’s unaffected ear.

 

 

18. The home health nurse is caring for a patient with tactile and visual deficits. The nurse is concerned about injury related to inability to feel harmful stimuli and teaches the patient safety strategies to maintain independence. Which action by the patient indicates successful learning?

a. Asks the nurse to test the temperature of the water before entering the bath.
b. Places colored stickers on the faucet handles to indicate temperature.
c. Replaces all lace-up shoes with Velcro straps for ease.
d. Uses a heating pad on a low setting to keep warm.

 

 

ANS: B
If a patient with tactile deficits also has a visual impairment, it is important to be sure that water faucets are clearly marked “hot” and “cold,” or use color codes (i.e., red for hot and blue for cold). Discourage the use of heating pads in this population. Asking the nurse to test the water does not promote independence, although it does promote safety. Velcro is easier for a patient with a tactile deficit to manipulate and promotes self-care but not safety.

19. A nurse is working to prevent blindness. Which preventive action is a priority?

a. Screen young adults early for visual impairments.
b. Include rubella and syphilis screening in the preconception care plan.
c. Instruct parents to report reduced eye contact from their child immediately.
d. Administer eye prophylactic antibiotics to newborns within 24 hours after birth.

 

 

ANS: B
Actions to prevent blindness must occur before vision impairment takes place. Screening for diseases such as rubella, syphilis, chlamydia, and gonorrhea that affect the development of vision in the fetus is a preventative measure. Vision testing after birth is important to begin steps to correct or identify the problem early on so the child can develop as normally as possible; waiting until children are young adults is too late. Another technique is administering eye prophylaxis in the form of erythromycin ointment approximately 1 hour after an infant’s birth. Reporting reduced eye contact is recommended but is not a preventative measure.

 

20. The nurse is caring for a patient in acute respiratory distress. The patient has multiple monitoring systems on that constantly beep and makes noise. The patient is becoming agitated and frustrated over the inability to sleep. Which action by the nurse is most appropriate for this patient?

a. Administer an opioid medication to help the patient sleep.
b. Keep the door open during the night.
c. Open the shades at night.
d. Provide the patient with earplugs.

 

 

ANS: D
Control of excessive stimuli becomes an important part of a patient’s care; earplugs provide relief. Quiet time means dimming the lights throughout the unit, closing the shades, and shutting the doors. Allow patients to shut their room door to decrease noise. Opioid medications (for pain relief) should not be the first option; however, antianxiety medications and sleep aids maybe
considered.

 

21. The nurse is caring for a patient diagnosed with expressive aphasia from a traumatic brain injury. Which goal will the nurse include in the plan of care?

a. Patient will carry a pen and a pad of paper around for communication.
b. Patient will recover full use of speech vocabulary in 1 day.
c. Patient will thicken drinks to prevent aspiration.
d. Patient will communicate nonverbally.

 

 

ANS: D
Expressive aphasia, a motor type of aphasia, is the inability to name common objects or express simple ideas in words or writing. To adapt to expressive aphasia, the nurse and the patient need to work on ways to communicate nonverbally through means such as pointing and gestures. Goals and outcomes need to be realistic and measurable; recovery in 1 day is not realistic. A patient who has expressive aphasia may not be able to speak or write words with a pen and paper. Thickening drinks prevent aspiration risk and is
not included in a plan of care for this patient.

 

22. The nurse caring for a group of patients is monitoring for sensory deprivation. Which patient will the nurse monitor most closely?

a. A patient in the ICU under constant monitoring following a myocardial infarction
b. A patient on the unit with tuberculosis on airborne precautions
c. A patient who recently had a stroke and has left-sided weakness
d. A patient receiving hospice care for end-stage lung cancer

 

 

ANS: B
A group at risk includes patients isolated in a health care setting or at home because of conditions such as active tuberculosis. Sensory deprivation occurs when a person has decreased stimulation and limited sensory input. A patient in isolation (airborne precautions) is at risk for sensory deprivation because of limited exposure to meaningful stimuli. A patient in the ICU would be at risk for sensory overload with all the monitors and visitors. A patient with a stroke may have difficulty with tactile sensation but is not at as high risk for sensory deprivation as is one in isolation. A patient with lung cancer may have deficits, but hospice is present so the patient is at home with others.

 

23. A nurse is caring for an older adult patient on bed rest with potential sensory deprivation. Which action will the nurse take?

a. Offer the patient a back rub.
b. Hang a “Do not disturb” sign on the patient’s door.
c. Ask the patient “Would you like a newspaper to read?”
d. Place the patient in the room farthest from the nurses’ station.

 

 

ANS: A
Comfort measures such as washing the face and hands and providing back rubs improve the quality of stimulation and lessen the chance of sensory deprivation. The patient with sensory deprivation needs meaningful stimuli, and therapeutic massage helps establish a humanistic relationship that the patient is missing. All of the other options do not promote patient-nurse interaction and promote further social isolation.

 

24. The nurse is caring for a patient who is a well-known surgeon at the hospital. The nurse notices the patient becoming more agitated and withdrawn with each group of surgeon visitors. The nurse and patient agree to place a “Do not disturb” sign on the door. A few hours later, the nurse notices a surgeon who is not involved in the patient’s care attempting to enter the room. Which response by the nurse is most appropriate?

a. Call for security to remove the surgeon.
b. Allow the surgeon to enter.
c. Firmly explain that the patient does not wish to have visitors at this time.
d. Scold the surgeon for not obeying the sign and respecting the patient’s wishes.

 

 

ANS: C
The nurse acts as an advocate for the patient (who is experiencing sensory overload and would benefit from a quiet environment) by firmly and politely asking the surgeon to leave regardless of position in the hospital. A creative solution to decrease excessive environmental stimuli that prevent restful, healing sleep is to institute “quiet time” in ICUs. Data collected from one hospital that implemented 1 hour of quiet time daily found decreased staff and unit noise and improved patient satisfaction. The nurse should not allow anyone to enter unless the patient approves it. Security is not a necessary measure at this time. The nurse should handle the situation with professionalism when addressing the surgeon; scolding the visitor is not appropriate.

25. The nurse is caring for a patient who is recovering from a traumatic brain injury and frequently becomes disoriented to everything except location. Which nursing intervention will the nurse add to the care plan to reduce confusion?

a. Keep a day-by-day calendar at the patient’s bedside.
b. Place a patient observer in the patient’s room for safety.
c. Assess the patient’s level of consciousness and document every 4 hours.
d. Prepare to discharge once the patient is awake, alert, and oriented.

 

 

ANS: A
Keeping a calendar in the patient’s room helps to orient the patient to the dates. In the home meaningful stimuli include pets, music, television, pictures of family members, and a calendar and clock. The same stimuli need to be present in health care settings. Assessing the patient’s level of consciousness is not an action that will directly affect the patient’s confusion. A patient observer is unnecessary unless the patient is in danger from the confusion. The nurse should encourage the patient toward recovery but should
be sensitive to the time it takes for progression.

 

26. A nurse establishing a relationship with the patient who is severely visually impaired is teaching the patient how to contact the nurse for assistance. Which action will the nurse take?

a. Place a raised Braille sticker on the call button.
b. Explain to the patient that a staff person will stop by once an hour to see if the patient needs anything.
c. Instruct the patient to tell a family member to get the attention of the staff.
d. Color code the nurse call system.

 

 

ANS: A
The nurse should devise a plan of care that is accommodating of the patient’s visual deficit. Placing a sticker on the nurse call system allows the patient to page the nurse for assistance as needed. Using family members is not the best option. Making hourly rounds is not sufficient; the nurse needs to ensure that the patient can get in touch at any time. Color coding the nurse call system
will not help a severely visually impaired patient.

 

27. The nurse is caring for a patient who is taking gentamicin for an infection. Which assessment is a priority?

a. Hearing
b. Vision
c. Smell
d. Taste

 

ANS: A
Some antibiotics (e.g., streptomycin, gentamicin, and tobramycin) are ototoxic and permanently damage the auditory nerve, whereas chloramphenicol sometimes irritates the optic nerve. Smell and taste are not as affected.

 

28. A nurse is teaching a patient about vision. In which order will the nurse describe the pathway for vision, beginning with the first structure?

1. Lens
2. Pupil
3. Retina
4. Cornea
5. Optic nerve
a. 2, 1, 4, 5, 3
b. 1, 2, 4, 3, 5
c. 4, 2, 1, 3, 5
d. 5, 2, 4, 1, 3

 

 

ANS: C
Light rays enter the convex cornea and begin to converge. An adjustment of light rays occurs as they pass through the pupil and lens. Change in the shape of the lens focuses light on the retina. The sensory retina contains the rods and cones (i.e., photoreceptor cells sensitive to stimulation from light). Photoreceptor cells send electrical potentials by way of the optic nerve to the brain.

 

29. A nurse is caring for a patient with a right hemisphere stroke and partial paralysis. Which action by assistive personnel (NAP) demonstrates an understanding of the needs of this patient?

a. Dressing the left side first
b. Dressing the right side first
c. Dressing the lower extremities first
d. Dressing the upper extremities first

 

 

ANS: A
Dressing the left side first will be reinforced by the nurse. If a patient has partial paralysis and reduced sensation, the patient dresses the affected side first; in this case, the left. A stroke in the right hemisphere affects the left side of the body. The right side or upper and lower extremities are not as effective.

MULTIPLE RESPONSE

1. A home care nurse is inspecting a patient’s house for safety issues. Which findings will cause the nurse to address the safety problems? (Select all that apply.)

a. Stairway faintly lit
b. Bathtub with grab bars
c. Scatter rugs in the kitchen
d. Absence of smoke alarms
e. Low-pile carpeting in the living room
f. Level thresholds between bathroom and bedroom

 

 

ANS: A, C, D
Assess the patient’s home for common hazards, including the following: (1) loose area rugs and runners placed over carpeting, (2) poor lighting in stairways, and (3) absence of smoke alarms. Because of reduced depth perception, patients can trip on throw rugs,  runners, or the edge of stairs. A bathtub with grab bars is safe and does not need to be addressed. Teach patients and family members to keep all flooring in good repair and advise them to use low-pile carpeting. Thresholds between rooms need to be level
with the floor.

TB Q&A 10th 48 Skin Integrity and Wound Care

Chapter 48: Skin Integrity and Wound Care

MULTIPLE CHOICE

1. A nurse participating in a research project associated with pressure ulcers will assess for what predisposing factor tends to
increase the risk for pressure ulcer development?
a. Decreased level of consciousness
b. Adequate dietary intake
c. Shortness of breath
d. Muscular pain
ANS: A
Patients who are confused or disoriented or who have changing levels of consciousness are unable to protect themselves. The
patient may feel the pressure but may not understand what to do to relieve the discomfort or to communicate that he or she is
feeling discomfort. Impaired sensory perception, impaired mobility, shear, friction, and moisture are other predisposing factors.
Shortness of breath, muscular pain, and an adequate dietary intake are not included among the predisposing factors.
DIF: Apply (application)
OBJ: Discuss the risk factors that contribute to pressure ulcer formation.
TOP: Assessment MSC: Reduction of Risk Potential
2. The nurse caring for an unconscious patient who was involved in an automobile accident 2 weeks ago will give priority to which
element when planning care to decrease the development of a decubitus ulcer?
a. Resistance
b. Pressure
c. Weight
d. Stress
ANS: B
Pressure is the main element that causes pressure ulcers. Three pressure-related factors contribute to pressure ulcer development:
pressure intensity, pressure duration, and tissue tolerance. When the intensity of the pressure exerted on the capillary exceeds 15 to
32 mm Hg, this occludes the vessel, causing ischemic injury to the tissues it normally feeds. High pressure over a short time and
low pressure over a long-time cause skin breakdown. Resistance, stress, and weight are not the priority causes of pressure ulcers.
DIF: Understand (comprehension)
OBJ: Discuss the risk factors that contribute to pressure ulcer formation.
TOP: Planning MSC: Reduction of Risk Potential
3. Which nursing observation will indicate the patient is at risk for pressure ulcer formation?
a. Fecal incontinence
b. Ate two thirds of breakfast
c. A raised red rash on the right shin
d. Capillary refill is less than 2 seconds
ANS: A
The presence and duration of moisture on the skin increase the risk of ulcer formation by making it susceptible to injury. Moisture
can originate from wound drainage, excessive perspiration, and fecal or urinary incontinence. Bacteria and enzymes in the stool can
enhance the opportunity for skin breakdown because the skin is moistened and softened, causing maceration. Eating a balanced diet
is important for nutrition, but eating just two thirds of the meal does not indicate that the individual is at risk. A raised red rash on
the leg again is a concern and can affect the integrity of the skin, but it is located on the shin, which is not a high-risk area for skin
breakdown. Pressure can influence capillary refill, leading to skin breakdown, but this capillary response is within normal limits.
DIF: Apply (application)
OBJ: Discuss the risk factors that contribute to pressure ulcer formation.
TOP: Assessment MSC: Reduction of Risk Potential
4. The wound care nurse is monitoring a patient with a Stage III pressure ulcer whose wound presents with healthy tissue. How
should the nurse document this ulcer in the patient’s medical record?
a. Stage I pressure ulcer
b. Healing Stage II pressure ulcer
c. Healing Stage III pressure ulcer
d. Stage III pressure ulcer
ANS: C
When a pressure ulcer has been staged and is beginning to heal, the ulcer keeps the same stage and is labeled with the words
“healing stage” or healing Stage III pressure ulcer. Once an ulcer has been staged, the stage endures even as the ulcer heals. This
ulcer was labeled a Stage III, and it cannot return to a previous stage such as Stage I or II. This ulcer is healing, so it is no longer
labeled a Stage III.
DIF: Apply (application) OBJ: Describe the pressure ulcer staging system.
TOP: Implementation MSC: Physiological Adaptation
Copyright © 2021, Elsevier Inc. All rights reserved. 2
5. The nurse admitting an older patient notes a shallow open reddish, pink ulcer without slough on the right heel of the patient. How
will the nurse stage this pressure ulcer?
a. Stage I
b. Stage II
c. Stage III
d. Stage IV
ANS: B
This would be a Stage II pressure ulcer because it presents as partial-thickness skin loss involving epidermis and dermis. The ulcer
presents clinically as an abrasion, blister, or shallow crater. Stage I is intact skin with nonblanchable redness over a bony
prominence. With a Stage III pressure ulcer, subcutaneous fat may be visible, but bone, tendon, and muscles are not exposed. Stage
IV involves full-thickness tissue loss with exposed bone, tendon, or muscle.
DIF: Apply (application) OBJ: Describe the pressure ulcer staging system.
TOP: Assessment MSC: Physiological Adaptation
6. Which item should the nurse use first to assist in staging an ulcer on the heel of a darkly pigmented skin patient?
a. Disposable measuring tape
b. Cotton-tipped applicator
c. Sterile gloves
d. Natural light
ANS: D
When assessing a patient with darkly pigmented skin, proper lighting is essential to accurately complete the first step in
assessment—inspection—and the entire assessment process. Natural light is recommended. Fluorescent light sources can produce
blue tones on darkly pigmented skin and can interfere with an accurate assessment. Other items that could possibly be used during
the assessment include gloves for infection control, a disposable measuring device to measure the size of the wound, and a
cotton-tipped applicator to measure the depth of the wound, but these items are not the first items used.
DIF: Understand (comprehension) OBJ: Describe the pressure ulcer staging system.
TOP: Assessment MSC: Health Promotion and Maintenance
7. The nurse is caring for a patient with a Stage IV pressure ulcer. Which type of healing will the nurse consider when planning care
for this patient?
a. Partial-thickness wound repair
b. Full-thickness wound repair
c. Primary intention
d. Tertiary intention
ANS: B
Stage IV pressure ulcers are full-thickness wounds that extend into the dermis and heal by scar formation because the deeper
structures do not regenerate, hence the need for full-thickness repair. The full-thickness repair has four phases: hemostasis,
inflammatory, proliferative, and maturation. A wound heals by primary intention when wounds such as surgical wounds have little
tissue loss; the skin edges are approximated or closed, and the risk for infection is low. Partial-thickness repairs are done on
partial-thickness wounds that are shallow, involving loss of the epidermis and maybe partial loss of the dermis. These wounds heal
by regeneration because the epidermis regenerates. Tertiary intention is seen when a wound is left open for several days, and then
the wound edges are approximated. Wound closure is delayed until risk of infection is resolved.
DIF: Apply (application) OBJ: Discuss the normal process of wound healing.
TOP: Planning MSC: Physiological Adaptation
8. The nurse is caring for a group of patients. Which patient will the nurse see first?
a. A patient with a Stage IV pressure ulcer
b. A patient with a Braden Scale score of 18
c. A patient with appendicitis using a heating pad
d. A patient with an incision that is approximated
ANS: C
The nurse should see the patient with an appendicitis first. Warm applications are contraindicated when the patient has an acute,
localized inflammation such as appendicitis because the heat could cause the appendix to rupture. Although a Stage IV pressure
ulcer is deep, it is not as critical as the appendicitis patient. The total Braden score ranges from 6 to 23; a lower total score indicates
a higher risk for pressure ulcer development. A score of 18 can be assessed later. A healing incision is approximated (closed); this
is a normal finding and does not need to be seen first.
DIF: Analyze (analysis)
OBJ: Complete an assessment for a patient with impaired skin integrity.
TOP: Assessment MSC: Management of Care
9. The nurse is caring for a patient who is experiencing a full-thickness wound repair. Which type of tissue will the nurse expect to
observe when the wound is healing?
a. Eschar
b. Slough
c. Granulation
d. Purulent drainage
ANS: C
Granulation tissue is red, moist tissue composed of new blood vessels, the presence of which indicates progression toward healing.
Soft yellow or white tissue is characteristic of slough—a substance that needs to be removed for the wound to heal. Black or brown
necrotic tissue is called eschar, which also needs to be removed for a wound to heal. Purulent drainage is indicative of an infection
and will need to be resolved for the wound to heal.
DIF: Apply (application) OBJ: Discuss the normal process of wound healing.
TOP: Assessment MSC: Physiological Adaptation
Copyright © 2021, Elsevier Inc. All rights reserved. 3
10. The nurse is caring for a patient who has experienced a laparoscopic appendectomy. For which type of healing will the nurse focus
the care plan?
a. Partial-thickness repair
b. Secondary intention
c. Tertiary intention
d. Primary intention
ANS: D
A clean surgical incision is an example of a wound with little loss of tissue that heals with primary intention. The skin edges are
approximated or closed, and the risk for infection is low. Partial-thickness repairs are done on partial-thickness wounds that are
shallow, involving loss of the epidermis and maybe partial loss of the dermis. These wounds heal by regeneration because the
epidermis regenerates. Tertiary intention is seen when a wound is left open for several days, and then the wound edges are
approximated. Wound closure is delayed until the risk of infection is resolved. A wound involving loss of tissue such as a burn or a
pressure ulcer or laceration heals by secondary intention. The wound is left open until it becomes filled with scar tissue. It takes
longer for a wound to heal by secondary intention; thus the chance of infection is greater.
DIF: Understand (comprehension)
OBJ: Describe the differences in wound healing by primary and secondary intention.
TOP: Planning MSC: Physiological Adaptation
11. The nurse caring for a patient in the burn unit should expect what type of wound healing when planning care for this patient?
a. Partial-thickness repair
b. Secondary intention
c. Tertiary intention
d. Primary intention
ANS: B
A wound involving loss of tissue such as a burn or a pressure ulcer or laceration heals by secondary intention. The wound is left
open until it becomes filled with scar tissue. It takes longer for a wound to heal by secondary intention; thus the chance of infection
is greater. A clean surgical incision is an example of a wound with little loss of tissue that heals by primary intention. The skin
edges are approximated or closed, and the risk for infection is low. Partial-thickness repair is done on partial-thickness wounds that
are shallow, involving loss of the epidermis and maybe partial loss of the dermis. These wounds heal by regeneration because the
epidermis regenerates. Tertiary intention is seen when a wound is left open for several days, and then the wound edges are
approximated. Wound closure is delayed until the risk of infection is resolved.
DIF: Understand (comprehension)
OBJ: Describe the differences in wound healing by primary and secondary intention.
TOP: Planning MSC: Physiological Adaptation
12. Which nursing observation will indicate the patient’s wound healed by the process of secondary intention?
a. Minimal loss of tissue function
b. Permanent dark redness at site
c. Minimal scar tissue
d. Scarring that may be severe
ANS: D
A wound healing by secondary intention takes longer than one healing by primary intention. The wound is left open until it
becomes filled with scar tissue. If the scarring is severe, permanent loss of function often occurs. Wounds that heal by primary
intention heal quickly with minimal scarring. Scar tissue contains few pigmented cells and has a lighter color than normal skin.
DIF: Understand (comprehension)
OBJ: Describe the differences in wound healing by primary and secondary intention.
TOP: Assessment MSC: Physiological Adaptation
13. The nurse is caring for a patient who has experienced a total abdominal hysterectomy. Which nursing observation related to the
incision will indicate the patient is experiencing a complication of wound healing?
a. Patient reporting, “My incision is hurting.”
b. Approximation of the incision edges has occurred.
c. Patient asks, “Why has my incision started to itch?”
d. The incision appears both swollen and bluish in color.
ANS: D
A hematoma is a localized collection of blood underneath the tissues. It appears as swelling, change in color, sensation, or warmth
or a mass that often takes on a bluish discoloration. A hematoma near a major artery or vein is dangerous because it can put
pressure on the vein or artery and obstruct blood flow. Itching is not a complication. Incisions should be approximated with edges
together; this is a sign of normal healing. After surgery, when nerves in the skin and tissues have been traumatized by the surgical
procedure, it is expected that the patient will experience pain.
DIF: Apply (application) OBJ: Describe complications of wound healing.
TOP: Assessment MSC: Reduction of Risk Potential
Copyright © 2021, Elsevier Inc. All rights reserved. 4
14. Which finding will alert the nurse to a potential wound dehiscence?
a. Protrusion of visceral organs through a wound opening
b. Chronic drainage of fluid through the incision site
c. Report by patient that something has given way
d. Drainage that is odorous and purulent
ANS: C
Patients often report feeling as though something has given way with dehiscence. Dehiscence occurs when an incision fails to heal
properly, and the layers of skin and tissue separate. It involves abdominal surgical wounds and occurs after a sudden strain such as
coughing, vomiting, or sitting up in bed. Evisceration is seen when vital organs protrude through a wound opening. When there is
an increase in serosanguineous drainage from a wound in the first few days after surgery, be alert for the potential for dehiscence.
Infection is characterized by drainage that is odorous and purulent.
DIF: Apply (application) OBJ: Describe complications of wound healing.
TOP: Assessment MSC: Reduction of Risk Potential
15. Which laboratory data will be important for the nurse to monitor when a patient develops a pressure ulcer?
a. Vitamin E
b. Potassium
c. Prealbumin
d. Sodium
ANS: C
Normal wound healing requires proper nutrition. Serum proteins are biochemical indicators of malnutrition, and serum albumin is
probably the most frequently measured of these parameters. The best measurement of nutritional status is prealbumin because it
reflects not only what the patient has ingested but also what the body has absorbed, digested, and metabolized. Zinc and copper are
the minerals important for wound healing, not potassium and sodium. Vitamins A and C are important for wound healing, not
vitamin E.
DIF: Apply (application)
OBJ: Explain the factors that impede or promote wound healing.
TOP: Assessment MSC: Reduction of Risk Potential
16. A nurse is caring for a patient with a wound. Which assessment data will be most relevant with regard to wound healing?
a. Muscular strength assessment
b. Pulse oximetry assessment
c. Sensation assessment
d. Sleep assessment
ANS: B
Oxygen fuels the cellular functions essential to the healing process; the ability to perfuse tissues with adequate amounts of
oxygenated blood is critical in wound healing. Pulse oximetry measures the oxygen saturation of blood. Assessment of muscular
strength and sensation, although useful for fitness and mobility testing, does not provide any data with regard to wound healing.
Sleep, although important for rest and for integration of learning and restoration of cognitive function, does not provide any data
with regard to wound healing.
DIF: Apply (application)
OBJ: Explain the factors that impede or promote wound healing.
TOP: Assessment MSC: Reduction of Risk Potential
17. Upon entering the room of a patient with a healing Stage III pressure ulcer, the nurse notices an odor and observes a purulent
discharge, along with increased redness at the wound site. What action should the nurse give priority to?
a. Completing a head-to-toe assessment, including current treatment, vital signs, and
laboratory results
b. Notifying the health care provider by utilizing Situation, Background,
Assessment, and Recommendation (SBAR)
c. Consulting the wound care nurse about the change in status and the potential for
infection
d. Conferring with the charge nurse about the change in status and the potential for
infection
ANS: A
The patient is showing signs and symptoms associated with infection in the wound. The nurse should complete the assessment:
gather all data such as current treatment modalities, medications, vital signs including temperature, and laboratory results such as
the most recent complete blood count or white cell count. The nurse can then notify the primary care provider and receive treatment
orders for the patient. It is important to notify the charge nurse and consult the wound nurse on the patient’s status and on any new
orders.
DIF: Apply (application)
OBJ: Explain the factors that impede or promote wound healing.
TOP: Implementation MSC: Reduction of Risk Potential
Copyright © 2021, Elsevier Inc. All rights reserved. 5
18. The nurse is collaborating with the dietitian about a patient with a Stage III pressure ulcer. Which nutrient will the nurse expect to
be increased after collaboration with the dietitian?
a. Fat
b. Protein
c. Vitamin E
d. Carbohydrate
ANS: B
Protein needs are especially increased in supporting the activity of wound healing. The physiological processes of wound healing
depend on the availability of protein, vitamins (especially A and C), and the trace minerals of zinc and copper. Wound healing does
not require increased amounts of fats or carbohydrates. Vitamin E will not be increased for wound healing.
DIF: Apply (application)
OBJ: Explain the factors that impede or promote wound healing.
TOP: Planning MSC: Management of Care
19. The nurse is completing an assessment on a patient who has a Stage IV pressure ulcer. The wound is odorous with a drain is
currently in place. Which statement by the patient indicates issues with self-concept?
a. “I am so weak and tired. I just want to feel better.”
b. “I been thinking I will be ready to go home early next week.”
c. “I really need a bath and linen change right; I feel so awful.”
d. “I am hoping there will be something good to eat for my dinner tonight.”
ANS: C
Body image changes can influence self-concept. The wound is odorous, and a drain is in place. The patient who is asking for a bath
and change in linens and states that this is awful gives you a clue that he or she may be concerned about the smell in the room.
Factors that affect the patient’s perception of the wound include the presence of scars, drains, odor from drainage, and temporary or
permanent prosthetic devices. The patient’s stating that he or she wants to feel better, talking about going home, and caring about
what is for dinner could be interpreted as positive statements that indicate progress along the health journey.
DIF: Analyze (analysis)
OBJ: Explain the factors that impede or promote wound healing.
TOP: Evaluation MSC: Psychosocial Integrity
20. A patient presents to the emergency department with a laceration of the right forearm caused by a fall. After determining that the
patient is stable, what is the next best step for the nurse to take?
a. Inspect the wound for bleeding.
b. Irrigate the wound to remove foreign bodies.
c. Measure and document the size of the wound.
d. Determine when the patient last had a tetanus antitoxin injection.
ANS: A
After determining that a patient’s condition is stable, inspect the wound for bleeding. An abrasion will have limited bleeding, a
laceration can bleed more profusely, and a puncture wound bleeds in relation to the size and depth of the wound. Address any
bleeding issues. Inspect the wound for foreign bodies; traumatic wounds are dirty and may need to be addressed. Determine the size
of the wound. A large open wound may expose bone or tissue and be protected, or the wound may need suturing. When the wound
is caused by a dirty penetrating object, determine the need for a tetanus vaccination.
DIF: Apply (application)
OBJ: Describe the differences between nursing care of acute and chronic wounds.
TOP: Implementation MSC: Management of Care
21. The nurse is caring for a patient on the medical-surgical unit with a wound that has a drain and a dressing that needs changing.
Which action should the nurse take first?
a. Provide analgesic medications as ordered.
b. Avoid accidentally removing the drain.
c. Don sterile gloves.
d. Gather supplies.
ANS: A
Because removal of dressings is painful, if often helps to give an analgesic at least 30 minutes before exposing a wound and
changing the dressing. The next sequence of events includes gathering supplies for the dressing change, donning gloves, and
avoiding the accidental removal of the drain during the procedure.
DIF: Apply (application)
OBJ: Describe the differences between nursing care of acute and chronic wounds.
TOP: Implementation MSC: Physiological Adaptation
Copyright © 2021, Elsevier Inc. All rights reserved. 6
22. The nurse is caring for a patient who has a wound drain with a collection device. The nurse notices that the collection device has a
sudden decrease in drainage. Which action will the nurse take initially?
a. Call the health care provider; a blockage is present in the tubing.
b. Chart the results on the intake and output flow sheet.
c. Do nothing, as long as the evacuator is compressed.
d. Remove the drain; a drain is no longer needed.
ANS: A
Because a drainage system needs to be patent, look for drainage flow through the tubing, as well as around the tubing. A sudden
decrease in drainage through the tubing may indicate a blocked drain, and you will need to notify the health care provider. The
health care provider, not the nurse, determines the need for drain removal and removes drains. Charting the results on the intake
and output flow sheet does not take care of the problem. The evacuator may be compressed even when a blockage is present.
DIF: Apply (application)
OBJ: Describe the differences between nursing care of acute and chronic wounds.
TOP: Implementation MSC: Reduction of Risk Potential
23. The nurse is caring for a patient who has a Stage IV pressure ulcer with grafted surgical sites. Which specialty bed will the nurse
use for this patient?
a. Low-air-loss
b. Air-fluidized
c. Lateral rotation
d. Standard mattress
ANS: B
For a patient with newly flapped or grafted surgical sites, the air-fluidized bed will be the best choice; this uses air and fluid support
to provide pressure redistribution via a fluid-like medium created by forcing air through beads as characterized by immersion and
envelopment. A low-air-loss bed is utilized for prevention or treatment of skin breakdown by preventing buildup of moisture and
skin breakdown through the use of airflow. A standard mattress is utilized for an individual who does not have actual or potential
altered or impaired skin integrity. Lateral rotation is used for treatment and prevention of pulmonary, venous stasis and urinary
complications associated with mobility.
DIF: Apply (application)
OBJ: Describe the differences between nursing care of acute and chronic wounds.
TOP: Planning MSC: Reduction of Risk Potential
24. The nurse notes that a patient has a black pressure ulcer on the left hip. Which event will the nurse anticipate when planning care
for this patient?
a. Increased monitoring of the wound condition
b. Documenting the wound’s status daily
c. Surgical debridement of the wound
d. Increased drainage from wound
ANS: C
Debridement is the removal of nonviable necrotic (black) tissue. Removal of necrotic tissue is necessary to rid the ulcer of a source
of infection, to enable visualization of the wound bed, and to provide a clean base for healing. A wound will not move through the
phases of healing if the wound is infected. When treating a pressure ulcer, it is important to monitor and reassess the wound at least
every 8 hours. Management of drainage will help keep the wound clean, but that is not the next step.
DIF: Apply (application)
OBJ: Describe the differences between nursing care of acute and chronic wounds.
TOP: Planning MSC: Physiological Adaptation
25. The nurse caring for a patient with a healing Stage III pressure ulcer notes that the wound is clean and granulating. Which health
care provider’s order will the nurse question?
a. Use a low-air-loss therapy unit.
b. Irrigate with Dakin’s solution.
c. Apply a hydrogel dressing.
d. Consult a dietitian.
ANS: B
Pressure ulcers should be with noncytotoxic cleansers such as normal saline, which will not kill fibroblasts and healing tissue.
Cytotoxic cleansers such as Dakin’s solution, acetic acid, povidone-iodine, and hydrogen peroxide can hinder the healing process
and should not be utilized on clean, granulating wounds. Consulting a dietitian for the nutritional needs of the patient, utilizing a
low-air-loss therapy unit to decrease pressure, and applying hydrogel dressings to provide a moist environment for healing are all
orders that would be appropriate.
DIF: Analyze (analysis)
OBJ: Describe the differences between nursing care of acute and chronic wounds.
TOP: Implementation MSC: Management of Care
Copyright © 2021, Elsevier Inc. All rights reserved. 7
26. The nurse is completing an assessment of the patient’s skin’s integrity. Which assessment is the priority?
a. Pressure points
b. Breath sounds
c. Bowel sounds
d. Pulse points
ANS: A
Observe pressure points such as bony prominences. The nurse continually assesses the skin for signs of ulcer development.
Assessment for tissue pressure damage includes visual and tactile inspection of the skin. Assessment of pulses, breath sounds, and
bowel sounds is part of a head-to-toe assessment and could influence the function of the body and ultimately skin integrity;
however, this assessment is not a specific part or priority of a skin assessment.
DIF: Apply (application)
OBJ: Complete an assessment for a patient with impaired skin integrity.
TOP: Assessment MSC: Management of Care
27. The nurse is completing a skin risk assessment using the Braden Scale. The patient has slight sensory impairment, has skin that is
rarely moist, walks occasionally, and has slightly limited mobility, along with excellent intake of meals and no apparent problem
with friction and shear. Which score will the nurse document for this patient?
a. 15
b. 17
c. 20
d. 23
ANS: C
With use of the Braden Scale, the total score is a 20. The patient receives 3 for slight sensory perception impairment, 4 for skin
being rarely moist, 3 for walks occasionally, 3 for slightly limited mobility, 4 for intake of meals, and 4 for no problem with
friction and shear.
DIF: Apply (application)
OBJ: Complete an assessment for a patient with impaired skin integrity.
TOP: Assessment MSC: Health Promotion and Maintenance
28. The nurse is caring for a surgical patient. Which intervention is most important for the nurse to complete to decrease the risk of
pressure ulcers and encourage the patient’s willingness and ability to increase mobility?
a. Explain the risks of immobility to the patient.
b. Turn the patient every 3 hours while in bed.
c. Encourage the patient to sit up in the chair.
d. Provide analgesic medication as ordered.
ANS: D
Maintaining adequate pain control (providing analgesic medications) and patient comfort increases the patient’s willingness and
ability to increase mobility, which in turn reduces pressure ulcer risks. Although sitting in the chair is beneficial, it does not
increase mobility or provide pain control. Explaining the risk of immobility is important for the patient because it may impact the
patient’s willingness but not his or her ability. Turning the patient is important for decreasing pressure ulcers but needs to be done
every 2 hours and, again, does not influence the patient’s ability to increase mobility.
DIF: Apply (application)
OBJ: List appropriate nursing interventions for a patient with impaired skin integrity.
TOP: Implementation MSC: Reduction of Risk Potential
29. The nurse is caring for a patient with a Stage IV pressure ulcer. Which nursing diagnosis should the nurse add to the care plan?
a. Readiness for enhanced nutrition
b. Impaired physical mobility
c. Impaired skin integrity
d. Chronic pain
ANS: C
After the assessment is completed and the information that the patient has a Stage IV pressure ulcer is gathered, a diagnosis of
Impaired skin integrity is selected. Readiness for enhanced nutrition would be selected for an individual with an adequate diet that
could be improved. Impaired physical mobility and Chronic pain do not support the current data in the question.
DIF: Apply (application)
OBJ: List nursing diagnoses associated with impaired skin integrity.
TOP: Planning MSC: Management of Care
30. The nurse documents the following assessment data: right heel with reddened area that does not blanch. Which nursing diagnosis
will the nurse assign to this patient?
a. Imbalanced nutrition: less than body requirements
b. Ineffective peripheral tissue perfusion
c. Risk for infection
d. Acute pain
ANS: B
The area on the heel has experienced a decreased supply of blood and oxygen (tissue perfusion), which has resulted in tissue
damage. The most appropriate nursing diagnosis with this information is Ineffective peripheral tissue perfusion. Risk for infection,
Acute pain, and Imbalanced nutrition do not support the data in the question.
DIF: Apply (application)
OBJ: List nursing diagnoses associated with impaired skin integrity.
TOP: Diagnosis MSC: Management of Care
Copyright © 2021, Elsevier Inc. All rights reserved. 8
31. The nurse caring for an immobile patient wants to decrease the risk of the formation of pressure ulcers. Which action will the nurse
take first?
a. Offer favorite fluids.
b. Turn the patient every 2 hours.
c. Determine the patient’s risk factors.
d. Encourage increased quantities of carbohydrates and fats.
ANS: C
The first step in prevention is to assess the patient’s risk factors for pressure ulcer development. When a patient is immobile, the
major risk to the skin is the formation of pressure ulcers. Nursing interventions focus on prevention. Offering favorite fluids,
turning, and increasing carbohydrates and fats are not the first steps. Determining risk factors is first, so interventions can be
implemented to reduce or eliminate those risk factors.
DIF: Apply (application)
OBJ: Complete an assessment for a patient with impaired skin integrity.
TOP: Implementation MSC: Health Promotion and Maintenance
32. Which health care team member will the nurse consult when a patient has received a nursing diagnosis of Impaired skin integrity?
a. Respiratory therapist
b. Registered dietitian
c. Case manager
d. Chaplain
ANS: B
Refer patients with pressure ulcers to the dietitian for early intervention for nutritional problems. Adequate calories, protein,
vitamins, and minerals promote wound healing for the impaired skin integrity. The nurse is the coordinator of care and
collaborating with the dietitian would result in planning the best meals for the patient. The respiratory therapist can be consulted
when a patient has issues with the respiratory system. Case management can be consulted when the patient has a discharge need. A
chaplain can be consulted when the patient has a spiritual need.
DIF: Apply (application)
OBJ: Develop a nursing care plan for a patient with impaired skin integrity.
TOP: Implementation MSC: Management of Care
33. When a comatose patient develops a Stage II pressure ulcer, the nurse includes the nursing diagnosis of Risk for infection to the
care plan. Which is the best goal for this patient?
a. The patient will state what to look for with regard to an infection.
b. The patient’s family will demonstrate specific care of the wound site.
c. The patient’s family members will wash their hands when visiting the patient.
d. The patient will remain free of odorous or purulent drainage from the wound.
ANS: D
Because the patient has an open wound and the skin is no longer intact to protect the tissue, the patient is at increased risk for
infection. The nurse will be assessing the patient for signs and symptoms of infection, including an increase in temperature, an
increase in white count, and odorous and purulent drainage from the wound. The patient is unconscious and is unable to
communicate the signs and symptoms of infection. It is important for the patient’s family to be able to demonstrate how to care for
the wound and wash their hands, but these statements are not goals or outcomes for this nursing diagnosis.
DIF: Apply (application)
OBJ: Develop a nursing care plan for a patient with impaired skin integrity.
TOP: Planning MSC: Management of Care
34. When caring for a group of patients, which task can the nurse delegate to the nursing assistive personnel (AP)?
a. Assessing a surgical patient for risk of pressure ulcers
b. Applying a gauze bandage to secure a nonsterile dressing
c. Treating a pressure ulcer on the buttocks of a medical patient
d. Implementing negative-pressure wound therapy on a stable patient
ANS: B
The skill of applying bandages to secure nonsterile dressings can be delegated to AP. Assessing pressure ulcer risk, treating a
pressure ulcer, and implementing negative-pressure wound therapy cannot be delegated to an AP.
DIF: Apply (application)
OBJ: List appropriate nursing interventions for a patient with impaired skin integrity.
TOP: Implementation MSC: Management of Care
Copyright © 2021, Elsevier Inc. All rights reserved. 9
35. The nurse performing a moist-to-dry dressing has prepared the supplies, solution, and removed the old dressing. In which order will
the nurse implement the following steps, starting with the first one?
1. Apply sterile gloves.
2. Cover and secure topper dressing.
3. Assess wound and surrounding skin.
4. Moisten gauze with prescribed solution.
5. Gently wring out excess solution and unfold.
6. Loosely pack until all wound surfaces are in contact with gauze.
a. 4, 3, 1, 5, 6, 2
b. 1, 3, 4, 5, 6, 2
c. 4, 1, 3, 5, 6, 2
d. 1, 4, 3, 5, 6, 2
ANS: B
The steps for a moist-to-dry dressing are as follows: (1) apply sterile gloves, (2) assess appearance of surrounding skin, (3) moisten
gauze with prescribed solution, (4) gently wring out excess solution and unfold; apply gauze as single layer directly onto wound
surface, (5) if wound is deep, gently pack dressing into wound base by hand until all wound surfaces are in contact with gauze, and
(6) cover with sterile dry gauze and secure topper dressing.
DIF: Understand (comprehension)
OBJ: List appropriate nursing interventions for a patient with impaired skin integrity.
TOP: Implementation MSC: Basic Care and Comfort
36. The nurse is caring for a patient at risk for skin impairment. Which initial action should the nurse take to decrease this risk?
a. After cleansing thoroughly dry the skin.
b. Request a therapeutic bed and mattress.
c. Pad the bed with absorbent pads.
d. Use products that retain moisture.
ANS: A
Use cleansers with nonionic surfactants that are gentle to the skin. After you clean the skin, make sure that it is completely dry.
Absorbent pads and garments are controversial and should be considered only when other alternatives have been exhausted.
Depending on the needs of the patient, a specialty bed may be needed, but again, this does not provide the initial defense for skin
breakdown. Use only products that wick moisture away from the patient’s skin.
DIF: Apply (application)
OBJ: Develop a nursing care plan for a patient with impaired skin integrity.
TOP: Implementation MSC: Reduction of Risk Potential
37. A patient at risk for skin impairment is able to sit up in a chair. How long should the nurse schedule the patient to sit in the chair?
a. Less than 2 hours at any one time
b. For a total of least than 3 hours daily
c. No longer than 30 minutes out of every hour
d. Until the patient expresses being uncomfortable
ANS: A
When patients are able to sit up in a chair, make sure to limit the amount of time to 2 hours or less at any given time. The chair
sitting time should be individualized. In the sitting position, pressure on the ischial tuberosities is greater than in a supine position.
Utilize foam, gel, or an air cushion to distribute weight. Sitting for longer than 2 hours can increase the chance of ischemia.
DIF: Apply (application)
OBJ: Develop a nursing care plan for a patient with impaired skin integrity.
TOP: Planning MSC: Reduction of Risk Potential
38. The nurse is caring for a patient who is immobile and is at risk for skin impairment. The plan of care includes turning the patient.
Which is the best method for repositioning the patient?
a. Place the patient in a 30-degree supine position.
b. Utilize a transfer device to lift the patient.
c. Elevate the head of the bed 45 degrees.
d. Slide the patient into the new position.
ANS: B
When repositioning the patient, obtain assistance and utilize a transfer device to lift rather than drag the patient. Sliding the patient
into the new position will increase friction. The patient should be placed in a 30-degree lateral position, not a supine position. The
head of the bed should be elevated less than 30 degrees to prevent pressure ulcer development from shearing forces.
DIF: Apply (application)
OBJ: List appropriate nursing interventions for a patient with impaired skin integrity.
TOP: Implementation MSC: Basic Care and Comfort
Copyright © 2021, Elsevier Inc. All rights reserved. 10
39. As prescribed, the nurse leaves the pressure ulcer open to air and does not apply a dressing. Which stage of ulcer did the nurse
appropriately treat?
a. A Stage I
b. A Stage II
c. A Stage III
d. A Stage IV
ANS: A
Stage I intact pressure ulcers that resolve slowly without epidermal loss over 7 to 14 days do not require a dressing. A composite
film, hydrocolloid, or hydrogel can be utilized on a clean Stage II. A hydrocolloid, hydrogel covered with foam, calcium alginate,
and gauze can be utilized with a clean Stage III. Hydrogel covered with foam, calcium alginate, and gauze can be utilized with a
clean Stage IV. An unstageable wound covered with eschar should utilize a dressing of adherent film or gauze with an ordered
solution of enzymes.
DIF: Understand (comprehension)
OBJ: List appropriate nursing interventions for a patient with impaired skin integrity.
TOP: Implementation MSC: Physiological Adaptation
40. The patient appears anxious as the nurse is preparing to change their wound dressing. Which action should the nurse take?
a. Distract the patient with the television.
b. Offer to explain what they should expect.
c. Suggest that the patient “Close your eyes.”
d. Wait until family is visiting to support the patient.
ANS: B
Explaining the procedure educates the patient regarding the dressing change and involves him in the care, thereby allowing the
patient some control in decreasing anxiety. Telling the patient to close the eyes and turning on the television are distractions that do
not usually decrease a patient’s anxiety. If the family is a support system, their presence may be helpful but it isn’t practical to
postpone the treatment until they are present.
DIF: Apply (application)
OBJ: List appropriate nursing interventions for a patient with impaired skin integrity.
TOP: Implementation MSC: Psychosocial Integrity
41. Which intervention should be included as the nurse cleanses a wound?
a. Allow the solution to flow from the most contaminated to the least contaminated.
b. Scrub vigorously when applying noncytotoxic solution to the skin.
c. Cleanse in a direction from the least contaminated area.
d. Utilize clean gauze and clean gloves to cleanse a site.
ANS: C
Cleanse in a direction from the least contaminated area, such as from the wound or incision, to the surrounding skin. While
cleansing surgical or traumatic wounds by applying noncytotoxic solution with sterile gauze or by irrigations is correct, vigorous
scrubbing is inappropriate and can cause damage to the skin. Use gentle friction when applying solutions to the skin and allow
irrigation to flow from the least to the most contaminated area.
DIF: Apply (application)
OBJ: List appropriate nursing interventions for a patient with impaired skin integrity.
TOP: Implementation MSC: Reduction of Risk Potential
42. Which is the best explanation for the nurse to provide when teaching the patient the reason for the binder after an open abdominal
aortic aneurysm repair?
a. It reduces edema at the surgical site.
b. It secures the dressing in place.
c. It immobilizes the abdomen.
d. It supports the abdomen.
ANS: D
The patient has a large abdominal incision. This incision will need support, and an abdominal binder will support this wound,
especially during movement, as well as during deep breathing and coughing. A binder can be used to immobilize a body part (e.g.,
an elastic bandage applied around a sprained ankle). A binder can be used to prevent edema, for example, in an extremity but in
this case is not used to reduce edema at a surgical site. A binder can be used to secure dressings such as elastic webbing applied
around a leg after vein stripping.
DIF: Understand (comprehension)
OBJ: List appropriate nursing interventions for a patient with impaired skin integrity.
TOP: Teaching/Learning MSC: Basic Care and Comfort
Copyright © 2021, Elsevier Inc. All rights reserved. 11
43. The nurse is caring for a postoperative patient recovering from a medial meniscus repair of the right knee. Which action should the
nurse take to assist with pain management?
a. Monitor vital signs every 15 minutes.
b. Check pulses in the right foot.
c. Keep the leg dependent.
d. Apply ice.
ANS: D
Ice assists in preventing edema formation, controlling bleeding, and anesthetizing the body part. Elevation (not dependent) assists
in preventing edema, which in turn can cause pain. Monitoring vital signs every 15 minutes is routine postoperative care and
includes a pain assessment but in itself is not an intervention that decreases pain. Checking the pulses is important to monitor the
circulation of the extremity but in itself is not a pain management intervention.
DIF: Apply (application)
OBJ: List appropriate nursing interventions for a patient with impaired skin integrity.
TOP: Implementation MSC: Basic Care and Comfort
44. The patient has a risk for skin impairment and has a 15 on the Braden Scale upon admission. The nurse has implemented
interventions. Upon reassessment, which Braden score will be the best sign that the risk for skin breakdown is removed?
a. 12
b. 13
c. 20
d. 23
ANS: D
The best sign is a perfect score of 23. The Braden Scale is composed of six subscales: sensory perception, moisture, activity,
mobility, nutrition, and friction and shear. The total score ranges from 6 to 23, and a lower total score indicates a higher risk for
pressure ulcer development. The cutoff score for onset of pressure ulcer risk with the Braden Scale in the general adult population
is 18.
DIF: Understand (comprehension)
OBJ: State evaluation criteria for a patient with impaired skin integrity.
TOP: Evaluation MSC: Reduction of Risk Potential
MULTIPLE RESPONSE
1. The nurse is caring for a patient with a surgical incision that eviscerates. Which actions will the nurse take? (Select all that apply.)
a. Place moist sterile gauze over the site.
b. Gently place the organs back.
c. Contact the surgical team.
d. Offer a glass of water.
e. Monitor for shock.
ANS: A, C, E
The presence of an evisceration (protrusion of visceral organs through a wound opening) is a surgical emergency. Immediately
place damp sterile gauze over the site, contact the surgical team, do not allow the patient anything by mouth (NPO), observe for
signs and symptoms of shock, and prepare the patient for emergency surgery.
DIF: Apply (application) OBJ: Describe complications of wound healing.
TOP: Implementation MSC: Physiological Adaptation
2. The nurse is caring for a patient with a wound healing by full-thickness repair. Which phases will the nurse monitor for in this
patient? (Select all that apply.)
a. Hemostasis
b. Maturation
c. Inflammatory
d. Proliferative
e. Reproduction
f. Reestablishment of epidermal layers
ANS: A, B, C, D
The four phases involved in the healing process of a full-thickness wound are hemostasis, inflammatory, proliferative, and
maturation. Three components are involved in the healing process of a partial-thickness wound: inflammatory response, epithelial
proliferation (reproduction) and migration, and reestablishment of the epidermal layers.
DIF: Understand (comprehension)
OBJ: Explain the factors that impede or promote wound healing.
TOP: Assessment MSC: Physiological Adaptation
Copyright © 2021, Elsevier Inc. All rights reserved. 12
3. The nurse is completing a skin assessment on a medical-surgical patient. Which nursing assessment questions should be included in
a skin integrity assessment? (Select all that apply.)
a. “Can you easily change your position?”
b. “Do you have sensitivity to heat or cold?”
c. “How often do you need to use the toilet?”
d. “What medications do you take?”
e. “Is movement painful?”
f. “Have you ever fallen?”
ANS: A, B, C, E
Changing positions is important for decreasing the pressure associated with long periods of time in the same position. If the patient
is able to feel heat or cold and is mobile, she can protect herself by withdrawing from the source. Knowing toileting habits and any
potential for incontinence is important because urine and feces in contact with the skin for long periods can increase skin
breakdown. Knowing whether the patient has problems with painful movement will alert the nurse to any potential for decreased
movement and increased risk for skin breakdown. Medications and falling are safety risk questions.
DIF: Apply (application)
OBJ: Complete an assessment for a patient with impaired skin integrity.
TOP: Assessment MSC: Reduction of Risk Potential
4. The nurse is caring for a patient with potential skin breakdown. Which components will the nurse include in the skin assessment?
(Select all that apply.)
a. Vision
b. Hyperemia
c. Induration
d. Blanching
e. Temperature of skin
ANS: B, C, D, E
Assessment of the skin includes both visual and tactile inspection. Assess for hyperemia and palpate for blanching or nonblaching.
Early signs of skin damage include induration, bogginess (less-than-normal stiffness), and increased warmth at the injury site
compared to nearby areas. Changes in temperature can indicate changes in blood flow to that area of the skin. Vision is not
included in the skin assessment.
DIF: Apply (application)
OBJ: Complete an assessment for a patient with impaired skin integrity.
TOP: Assessment MSC: Health Promotion and Maintenance
5. The nurse is caring for a patient who will have a large abdominal bandage secured with an abdominal binder. Which actions will
the nurse take before applying the bandage and binder? (Select all that apply.)
a. Cover exposed wounds.
b. Mark the sites of all abrasions.
c. Assess the condition of current dressings.
d. Inspect the skin for abrasions and edema.
e. Cleanse the area with hydrogen peroxide.
f. Assess the skin at underlying areas for circulatory impairment.
ANS: A, C, D, F
Before applying a bandage or a binder, the nurse has several responsibilities. The nurse would need to inspect the skin for
abrasions, edema, and discoloration or exposed wound edges. The nurse also is responsible for covering exposed wounds or open
abrasions with a dressing and assessing the condition of underlying dressings and changing if soiled, as well as assessing the skin
of underlying areas that will be distal to the bandage. This checks for signs of circulatory impairment, so that a comparison can be
made after bandages are applied. Marking the sites of all abrasions is not necessary. Although it is important for the skin to be
clean, and even though it may need to be cleaned with a noncytotoxic cleanser, cleansing with hydrogen peroxide can interfere with
wound healing.
DIF: Apply (application)
OBJ: List appropriate nursing interventions for a patient with impaired skin integrity.
TOP: Implementation MSC: Basic Care and Comfort
6. The nurse is updating the plan of care for a patient with impaired skin integrity. Which findings indicate achievement of goals and
outcomes? (Select all that apply.)
a. The patient’s expectations are not being met.
b. Skin is intact with no redness or swelling.
c. Nonblanchable erythema is absent.
d. No injuries to the skin and tissues are evident.
e. Granulation tissue is present.
ANS: B, C, D, E
Optimal outcomes are to prevent injury to skin and tissues, reduce injury to skin, reduce injury to underlying tissues, and restore
skin integrity. Skin intact, nonblanchable erythema absent, no injuries, and presence of granulation tissue are all findings indicating
achievement of goals and outcomes. The patient’s expectations not being met indicate no progression toward goals/outcomes.
DIF: Analyze (analysis)
OBJ: State evaluation criteria for a patient with impaired skin integrity.
TOP: Evaluation MSC: Management of Care
Copyright © 2021, Elsevier Inc. All rights reserved. 13
MATCHING
The nurse is caring for patients who need wound dressings. Match the type of dressing the nurse applies to its description.
a. Absorbs drainage through the use of exudate absorbers in the dressing
b. Very soothing to the patient and do not adhere to the wound bed
c. Barrier to external fluids/bacteria but allows wound to “breathe”
d. Manufactured from seaweed and comes in sheet and rope form
e. Oldest and most common absorbent dressing
1. Gauze
2. Transparent
3. Hydrocolloid
4. Hydrogel
5. Calcium alginate
1. ANS: E DIF: Understand (comprehension)
OBJ: List appropriate nursing interventions for a patient with impaired skin integrity.
TOP: Implementation MSC: Basic Care and Comfort
2. ANS: C DIF: Understand (comprehension)
OBJ: List appropriate nursing interventions for a patient with impaired skin integrity.
TOP: Implementation MSC: Basic Care and Comfort
3. ANS: A DIF: Understand (comprehension)
OBJ: List appropriate nursing interventions for a patient with impaired skin integrity.
TOP: Implementation MSC: Basic Care and Comfort
4. ANS: B DIF: Understand (comprehension)
OBJ: List appropriate nursing interventions for a patient with impaired skin integrity.
TOP: Implementation MSC: Basic Care and Comfort
5. ANS: D DIF: Understand (comprehension)
OBJ: List appropriate nursing interventions for a patient with impaired skin integrity.
TOP: Implementation MSC: Basic Care and Comfort

TB Q&A 10th 43 Sleep

Chapter 43: Sleep

MULTIPLE CHOICE

1. The nurse is caring for a young-adult patient on the medical-surgical unit. When doing midnight checks, the nurse observes the
patient awake, putting a puzzle together. Which information will the nurse consider to best explain this finding?
a. The patient misses family and is lonely.
b. The patient was waiting to talk with the nurse.
c. The patient has been kept up with the noise on the unit.
d. The patient’s sleep-wake cycle preference is late evening.
ANS: D
This patient is awake and alert enough to do a puzzle. The individual’s sleep-wake preference is probably late evening. All persons
have biological clocks that synchronize their sleep-wake cycle. This explains why some individuals fall asleep in the early evening,
whereas others go to bed at midnight or early morning. Waiting to talk with the nurse, being lonely, and noise on the unit may
contribute to lack of sleep, but the best explanation for the patient being awake is the biological clock.
DIF: Analyze (analysis)
OBJ: Explain the effect the 24-hour sleep-wake cycle has on biological function.
TOP: Evaluation MSC: Basic Care and Comfort
2. The nurse is providing an educational session on sleep regulation for new nurses in the Sleep Disorder Treatment Center. Which
statement by the nurses will best indicate that the teaching is effective?
a. “If the patient has a disease process in the central nervous system, it can influence
the functions of sleep.”
b. “If the patient has a disease process in the cranial nerves, it can influence the
functions of sleep.”
c. “If the patient has an interruption in the urinary pathways, it can influence the
functions of sleep.”
d. “If the patient has an interruption in the spinal reflexes, it can influence the
functions of sleep.”
ANS: A
Sleep involves a sequence of physiological states maintained by the central nervous system. It is associated with changes in the
peripheral nervous, endocrine, cardiovascular, respiratory, and muscular systems. A disease process associated with the cranial
nerves, urinary pathway, or spinal reflexes may influence a person’s ability to sleep, but the best answer is the central nervous
system.
DIF: Analyze (analysis) OBJ: Discuss mechanisms that regulate sleep.
TOP: Teaching/Learning MSC: Physiological Adaptation
3. The nurse is caring for a patient who reports having trouble sleeping. Which action will the nurse take?
a. Suggest snug-fitting nightwear.
b. Provide a favorite beverage.
c. Encourage deep breathing.
d. Walk with the patient.
ANS: C
Relaxation exercises such as slow, deep breathing for 1 or 2 minutes relieve tension and prepare the body for rest. Instruct patients
to wear loose-fitting nightwear. Walking and drinking a favorite beverage would not necessarily encourage sleep.
DIF: Apply (application) OBJ: Discuss mechanisms that regulate sleep.
TOP: Implementation MSC: Basic Care and Comfort
4. The nurse is caring for a patient in the sleep lab. Which assessment finding indicates to the nurse that the patient is in stage 4
NREM?
a. The patient awakens easily.
b. The patient’s eyes rapidly move.
c. The patient is difficult to awaken.
d. The patient’s vital signs are elevated.
ANS: C
The patient is difficult to arouse, vital signs are significantly lower, and this stage lasts about 15 to 30 minutes. Stage 4 NREM is
the deepest stage of sleep. Lighter sleep is seen in stages 1 and 2, where the patient awakens easily. REM sleep is characterized by
rapid eye movement.
DIF: Apply (application) OBJ: Describe the stages of a normal sleep cycle.
TOP: Assessment MSC: Basic Care and Comfort
5. A nurse is teaching the staff about the sleep cycle. Which sequence will the nurse include in the teaching session?
a. NREM Stage 1, 2, 3, 4, REM
b. NREM Stage 1, 2, 3, 4, 3, 2, 1, REM
c. NREM Stage 1, 2, 3, 4, REM, 4, 3, 2 REM
d. NREM Stage 1, 2, 3, 4, 3, 2, REM
ANS: D
The cyclical pattern usually progresses from stage 1 through stage 4 of NREM, followed by a reversal from stages 4 to 3 to 2,
ending with a period of REM sleep. The others are incorrect sequences.
DIF: Understand (comprehension) OBJ: Describe the stages of a normal sleep cycle.
TOP: Teaching/Learning MSC: Physiological Adaptation
Copyright © 2021, Elsevier Inc. All rights reserved. 2
6. Which nursing observation of the patient in intensive care indicates the patient is sleeping comfortably during NREM sleep?
a. Eyes closed, lying quietly, respirations 12, heart rate 60
b. Eyes closed, tossing in bed, respirations 18, heart rate 80
c. Eyes closed, mumbling to self, respirations 16, heart rate 68
d. Eyes closed, lying supine in bed, respirations 22, heart rate 66
ANS: A
During NREM sleep, biological functions slow. During sleep, the heart rate decreases to 60 beats/min or less. The patient
experiences decreased respirations, blood pressure, and muscle tone. Heart rates above 60 are too high and respirations of 22 are
too high to indicate comfortable NREM sleep.
DIF: Analyze (analysis) OBJ: Explain the functions of sleep.
TOP: Assessment MSC: Basic Care and Comfort
7. The nurse is teaching a new mother about the sleep requirements of a neonate. Which comment by the patient indicates a correct
understanding of the teaching?
a. “I can’t wait to get the baby home to play with the brothers and sisters.”
b. “I will ask my mom to come after the first week, when the baby is more alert.”
c. “I can get the baby on a sleeping schedule the first week while my mom is here.”
d. “I won’t be able to nap during the day because the baby will be awake.”
ANS: B
The patient indicates an understanding when asking the mother to come after the first week. The neonate up to the age of 3 months
averages about 16 hours of sleep a day, sleeping almost constantly during the first week. The baby will sleep rather than play. The
baby will not be on a sleeping schedule the first week home. The mother will be able to nap since the baby sleeps 16 hours a day.
DIF: Analyze (analysis)
OBJ: Compare and contrast the sleep requirements of different age-groups.
TOP: Teaching/Learning MSC: Health Promotion and Maintenance
8. The nurse is discussing lack of sleep with a middle-aged adult. Which area should the nurse most likely assess to determine a
possible cause of the lack of sleep?
a. Anxiety
b. Loud teenagers
c. Caring for pets
d. Late night television
ANS: A
During middle adulthood, the total time spent sleeping at night begins to decline. Anxiety, depression, and certain physical illnesses
can affect sleep, and women can experience menopausal symptoms. Insomnia is common because of the changes and stresses
associated with middle age. Teenagers, caring for pets, and late-night television can influence the amount of sleep; however, these
are not the most common causes of insomnia in this age-group.
DIF: Apply (application)
OBJ: Compare and contrast the sleep requirements of different age-groups.
TOP: Assessment MSC: Basic Care and Comfort
9. A parent is discussing the sleep needs of a preschooler with the nurse. Which information will the nurse share with the parent?
a. “Most preschoolers sleep soundly all night long.”
b. “It is important that the 5 year old get a nap every day.”
c. “On average, the preschooler needs to sleep 10 hours a night.”
d. “Preschoolers may have trouble settling down after a busy day.”
ANS: D
The preschooler usually has difficulty relaxing or settling down after long, active days. By the age of 5, naps are rare for children,
except those for whom a siesta is a custom. Preschoolers frequently awaken during the night. On average, a preschooler needs
about 12 hours of sleep.
DIF: Understand (comprehension)
OBJ: Compare and contrast the sleep requirements of different age-groups.
TOP: Teaching/Learning MSC: Health Promotion and Maintenance
10. The nurse is having a conversation with an adolescent regarding the need for sleep. The adolescent states that it is common to stay
up with friends several nights a week. Which action should the nurse take next?
a. Talk with the adolescent’s parent about staying up with friends and the need for
sleep.
b. Discuss with the adolescent sleep needs and the effects of excessive daytime
sleepiness.
c. Refer the adolescent for counseling about alcohol abuse problems.
d. Take no action for this normal occurrence.
ANS: B
Discussion regarding adolescent sleep needs should first occur with the adolescent. Although it may be common for this adolescent
to want to visit with friends and experience activities that go late into the night, these activities can and do impact the hours of sleep
and the physical needs of the adolescent, no matter the reason for the late nights, and they do need to be addressed. The nurse will
address the adolescent, not the parents. Addressing alcohol abuse problems is not the next step but may be required later. While
staying up late may be a normal occurrence for this adolescent, action is required.
DIF: Apply (application)
OBJ: Compare and contrast the sleep requirements of different age-groups.
TOP: Implementation MSC: Health Promotion and Maintenance
Copyright © 2021, Elsevier Inc. All rights reserved. 3
11. The nurse is completing an assessment on an older-adult patient who is having difficulty falling asleep. Which condition will the
nurse further assess for in this patient?
a. Depression
b. Mild fatigue
c. Hypertension
d. Hypothyroidism
ANS: A
Older adults and other individuals who experience depressive mood problems experience delays in falling asleep, earlier
appearance of REM sleep, frequent awakening, feelings of sleeping poorly, and daytime sleepiness. A person who is moderately
fatigued usually achieves restful sleep, especially if the fatigue is the result of enjoyable work or exercise. Hypertension often
causes early-morning awakening and fatigue. Alcohol speeds the onset of sleep. Hypothyroidism decreases stage 4 sleep.
DIF: Apply (application)
OBJ: Identify factors that normally promote and disrupt sleep. TOP: Assessment
MSC: Basic Care and Comfort
12. The nurse is caring for an adolescent post-appendectomy who is reporting difficulty falling asleep. Which intervention will be most
appropriate?
a. Close the door to decrease noise from unit activities.
b. Adjust temperature in the patient’s room to 21° C (70° F).
c. Ensure that the night-light in the patient’s room is working.
d. Encourage the discontinuation of a soda and chocolate as a nightly snack.
ANS: D
Discontinuing the soda and chocolate nightly snack will be most beneficial for this patient since it has two factors that will cause
difficulty falling asleep. Coffee, tea, colas, and chocolate act as stimulants, causing a person to stay awake or to awaken throughout
the night. Personal preference influences the temperature of the room, as well as the lighting of the room. Noise can be a factor in
the unit and can awaken the patient, but caffeine can make it difficult to fall asleep.
DIF: Analyze (analysis)
OBJ: Identify factors that normally promote and disrupt sleep. TOP: Implementation
MSC: Basic Care and Comfort
13. A patient is diagnosed with obstructive sleep apnea. Which assessment is the priority?
a. Gastrointestinal function
b. Neurological function
c. Respiratory status
d. Circulatory status
ANS: C
In obstructive sleep apnea, the upper airway becomes partially or completely blocked, diminishing nasal airflow or stopping it. The
person still attempts to breathe because the chest and abdominal movement continue, which results in loud snoring and snorting
sounds. According to the ABCs of prioritizing care, airway and respiratory status takes priority over gastrointestinal, circulatory,
and neurological functioning.
DIF: Apply (application)
OBJ: Discuss the characteristics of common sleep disorders. TOP: Assessment
MSC: Management of Care
14. The patient has just been diagnosed with narcolepsy. The nurse teaches the patient about management of the condition. Which
information from the patient will cause the nurse to intervene?
a. Takes antidepressant medications.
b. Naps shorter than 20 minutes.
c. Sleeps in hot, stuffy room.
d. Chews gum regularly.
ANS: C
The nurse will intervene about sitting in a hot, stuffy room as this will make the narcolepsy worse so this needs to be corrected.
Patients with narcolepsy need to avoid factors that increase drowsiness (e.g., alcohol, heavy meals, exhausting activities,
long-distance driving, and long periods of sitting in hot, stuffy rooms). Patients are treated with antidepressants, and management
techniques involve scheduling naps no longer than 20 minutes and chewing gum. Additional management techniques include
exercise, light high-protein meals, deep breathing, and taking vitamins.
DIF: Apply (application)
OBJ: Discuss the characteristics of common sleep disorders. TOP: Teaching/Learning
MSC: Physiological Adaptation
15. The nurse is caring for a patient who has been in holding in the emergency department for 24 hours. The nurse is concerned about
the patient’s experiencing sleep deprivation. Which action will be best for the nurse to take?
a. Expedite the process of obtaining a medical-surgical room for the patient.
b. Pull the curtains shut, dim the lights, and decrease the number of visitors.
c. Obtain an order for a hypnotic medication to help the patient sleep.
d. Ask everyone in the unit to try to be quiet so the patient can sleep.
ANS: A
The most effective treatment for sleep deprivation is elimination or correction of factors that disrupt the sleep pattern. Obtaining a
private room in the medical-surgical unit for the patient will help with decreasing stimuli and promoting more rest than an
individual can obtain in an emergency department even with the interventions mentioned.
DIF: Analyze (analysis) OBJ: Discuss characteristics of common sleep disorders.
TOP: Implementation MSC: Management of Care
Copyright © 2021, Elsevier Inc. All rights reserved. 4
16. The nurse is completing a sleep assessment on a patient. Which tool will the nurse use to complete the assessment?
a. Visual analog scale
b. Cataplexy scale
c. Polysomnogram
d. RAS scale
ANS: A
The visual analog scale is utilized for assessing sleep quality. Cataplexy, or sudden muscle weakness during intense emotions such
as anger, sadness, or laughter, occurs at any time during the day; there is no cataplexy scale for sleep assessment. A
polysomnogram involves the use of EEG, EMG, and EOG to monitor stages of sleep and wakefulness during nighttime sleep; this
is used in a sleep laboratory study. Researchers believe that the ascending reticular activating system (RAS) located in the upper
brainstem contains special cells that maintain alertness and wakefulness; however, there is no assessment tool called the RAS scale.
DIF: Understand (comprehension) OBJ: Conduct a sleep history for a patient.
TOP: Assessment MSC: Basic Care and Comfort
17. The nurse is beginning a sleep assessment on a patient. Which question will be most appropriate for the nurse to ask initially?
a. “What is going on?”
b. “How are you sleeping?”
c. “Are you taking any medications?”
d. “What did you have for dinner last night?”
ANS: B
Sleep is a subjective experience. Only the patient is able to report whether or not it is sufficient and restful. Asking patients how
they are sleeping is an introductory question. After this beginning question is asked, problems with sleep such as the nature of the
problem, signs and symptoms, onset and duration of the issue, severity, predisposing factors, and the effect on the patient can be
assessed. What is going on is too broad and open ended for information about sleep to be obtained specifically. Medications and
food intake can be part of the detailed assessment of sleep issues.
DIF: Apply (application) OBJ: Conduct a sleep history for a patient.
TOP: Assessment MSC: Basic Care and Comfort
18. The nurse adds a nursing diagnosis of Ineffective Breathing Pattern to a patient’s care plan. Which sleep condition likely caused
the nurse to assign this nursing diagnosis?
a. Insomnia
b. Narcolepsy
c. Sleep deprivation
d. Obstructive sleep apnea
ANS: D
Obstructive sleep apnea (OSA) occurs when the muscles or structures of the oral cavity or throat relax during sleep. The upper
airway becomes partially or completely blocked, diminishing airflow or stopping it for as long as 30 seconds. The person still
attempts to breathe because chest and abdominal movements continue, resulting in snoring or snorting sounds. With narcolepsy, the
person feels an overwhelming wave of sleepiness and falls asleep. Insomnia is characterized by chronic difficulty falling asleep.
Sleep deprivation is a condition caused by dyssomnia. OSA is the only one of these conditions that results in blockage of the
airway and impacts the ability to breathe.
DIF: Apply (application)
OBJ: Identify nursing diagnoses appropriate for patients with sleep alterations.
TOP: Diagnosis MSC: Physiological Adaptation
19. The nurse is caring for a postpartum patient whose labor lasted over 28 hours. The patient has not slept since delivering and is
disoriented to date and time. Which nursing diagnosis will the nurse document in the patient’s care plan?
a. Insomnia
b. Impaired parenting
c. Ineffective coping
d. Sleep deprivation
ANS: D
This patient has been deprived of sleep by staying awake during a 28-hour labor. Disorientation is one potential sign of sleep
deprivation. In this scenario, there is a clear cause for the patient’s lack of sleep, and it is a one-time episode. Insomnia, on the
other hand, is a chronic disorder whereby patients have difficulty falling asleep, awaken frequently, or sleep only for a short time.
This scenario does not indicate that this has been a chronic problem for this patient. Although ineffective coping can manifest as a
sleep disturbance, clear evidence shows that it was labor that deprived this patient of sleep, not an inability to cope. It could be
difficult to care for an infant when sleep deprived; however, this scenario gives no evidence that this mother displays impaired
parenting and is not caring adequately for her child or lacks the skills to do so.
DIF: Analyze (analysis)
OBJ: Identify nursing diagnoses appropriate for patients with sleep alterations.
TOP: Diagnosis MSC: Management of Care
Copyright © 2021, Elsevier Inc. All rights reserved. 5
20. The patient presents to the clinic with reports of irritability and anxiety, being sleepy during the day, chronically not being able to
fall asleep, and being fatigued. Which nursing diagnosis will the nurse document in the plan of care?
a. Anxiety
b. Fatigue
c. Insomnia
d. Sleep deprivation
ANS: C
Insomnia is experienced when the patient has chronic difficulty falling asleep, frequent awakenings from sleep, and/or short sleep
or nonrestorative sleep. It is the most common sleep-related complaint and includes symptoms such as irritability, excessive
daytime sleepiness, not being able to fall asleep, and fatigue. Anxiety is a vague, uneasy feeling of discomfort or dread
accompanied by an autonomic response. Fatigue is an overwhelming sustained sense of exhaustion with decreased capacity for
physical and mental work at a usual level. Sleep deprivation is a condition caused by dyssomnia and includes symptoms caused by
illness, emotional distress, or medications.
DIF: Apply (application)
OBJ: Identify nursing diagnoses appropriate for patients with sleep alterations.
TOP: Diagnosis MSC: Management of Care
21. The nurse is preparing an older-adult patient’s evening medications. Which treatment will the nurse recognize as relatively safe for
difficulty sleeping in older adults?
a. Ramelteon
b. Benzodiazepine
c. Antihistamine
d. Kava
ANS: A
Ramelteon, a melatonin receptor agonist, is well tolerated and appears to be effective in improving sleep by improving the
circadian rhythm and shortening time to sleep onset. It is safe for long- and short-term use particularly in older adults. The use of
benzodiazepines in older adults is potentially dangerous because of the tendency of the drugs to remain active in the body for a
longer time. As a result, they also cause respiratory depression, next-day sedation, amnesia, rebound insomnia, and impaired motor
functioning and coordination, which leads to increased risk of falls. Caution older adults about using over-the-counter
antihistamines because their long duration of action can cause confusion, constipation, and urinary retention. Kava promotes sleep
in patients with anxiety; it should be used cautiously because of its potential toxic effects on the liver.
DIF: Apply (application)
OBJ: Identify nursing interventions designed to promote normal sleep cycles for patients of all ages.
TOP: Implementation MSC: Pharmacological and Parenteral Therapies
22. The nurse is caring for a patient on the medical-surgical unit who is experiencing an exacerbation of asthma. Which intervention
will be most appropriate to help this patient sleep?
a. Place bed in semi-Fowler’s position.
b. Offer iron-rich foods for meals.
c. Provide a snack before bedtime.
d. Encourage the patient to read.
ANS: A
Placing the patient in a semi-Fowler’s position eases the work of breathing. Respiratory disease often interferes with sleep. Patients
with chronic lung disease such as emphysema or asthma are short of breath and frequently cannot sleep without two or three
pillows to raise their heads. Iron-rich food may help a patient with restless legs syndrome. Providing a snack and encouraging the
patient to read may be good interventions for patients, but the most appropriate would be raising the head of the bed.
DIF: Apply (application)
OBJ: Identify nursing interventions designed to promote normal sleep cycles for patients of all ages.
TOP: Implementation MSC: Basic Care and Comfort
23. A young adult has been hospitalized for an irregular heartbeat (dysrhythmia). The night nurse makes rounds and finds the patient
awake. Which action by the nurse is most appropriate?
a. Inform the patient that it is late and time to go to sleep.
b. Ask the patient if he/she would like medication for sleep.
c. Recommend a good movie that is on television tonight.
d. Take time to sit and talk with the patient about his/her inability to sleep.
ANS: D
A nurse on the night shift needs to take time to sit and talk with patients unable to sleep. This helps to determine the factors keeping
patients awake. Assessment is the first step of the nursing process; therefore, assessment needs to be done first and involves
ascertaining the cause of the patient’s inability to sleep. Patients who are admitted to the hospital for uncertain diagnoses can be
stressed and worried about the testing and outcomes. In addition, a young mother can be worried about the care of her children and
those caring for the children. This uncertainty and change in routine can cause difficulty in resting or falling asleep. A distraction
such as a television may or may not work for the patient. After assessment is completed, a sedative may or may not be in order.
Telling the patient that it is late and time to go to sleep is not a therapeutic response for an adult who is under stress.
DIF: Apply (application)
OBJ: Identify nursing interventions designed to promote normal sleep cycles for patients of all ages.
TOP: Implementation MSC: Psychosocial Integrity
Copyright © 2021, Elsevier Inc. All rights reserved. 6
24. The nurse is evaluating outcomes for the patient diagnosed with insomnia. Which key principle will the nurse consider during this
process?
a. The patient is the best evaluator of sleep.
b. The nurse is the best evaluator of sleep.
c. Effective interventions are the best evaluators of sleep.
d. Observations of the patient are the best evaluators of sleep.
ANS: A
With regard to problems with sleep, the patient is the source for evaluating outcomes. The patient is the only one who knows
whether sleep problems have improved and what has been successful. Interventions are not the best indicator; achievement of goals
according to the patient is the best. Observations do provide needed data, but in the case of insomnia, the patient is the source for
evaluating the restfulness of sleep.
DIF: Understand (comprehension)
OBJ: Describe ways to evaluate the effects of sleep therapies. TOP: Evaluation
MSC: Basic Care and Comfort
25. A patient is experiencing sleep deprivation. Which statement by the patient will indicate to the nurse that outcomes are being met?
a. “I wake up only once a night to go to the bathroom.”
b. “I feel rested when I wake up in the morning.”
c. “I go to sleep within 30 minutes of lying down.”
d. “I only take a 20-minute nap during the day.”
ANS: B
Being able to sleep and feeling rested would indicate that outcomes are being met for sleep deprivation. Limiting a nap to 20
minutes is an intervention to promote sleep. Going to sleep within 30 minutes indicates a goal for insomnia. Waking up only once
may indicate nocturia is improving but does not relate to sleep deprivation.
DIF: Analyze (analysis)
OBJ: Describe ways to evaluate the effects of sleep therapies. TOP: Evaluation
MSC: Basic Care and Comfort
26. An older-adult patient is visiting the clinic after a fall during the night. The nurse obtains information on what medications the
patient takes. Which medication most likely contributed to the patient’s fall?
a. Melatonin
b. L-tryptophan
c. Benzodiazepine
d. Iron supplement
ANS: C
The most likely cause is a benzodiazepine. If older patients who were recently continent, ambulatory, and alert become incontinent
or confused and/or demonstrate impaired mobility, the use of benzodiazepines needs to be considered as a possible cause. This can
contribute to a fall in an older adult. Short-term use of melatonin has been found to be safe, with mild side effects of nausea,
headache, and dizziness being infrequent. Iron supplements may be given to patients with restless legs syndrome. Some substances
such as L-tryptophan, a natural protein found in foods such as milk, cheese, and meats, promote sleep; while it does promote sleep,
it is not the most likely to cause mobility problems.
DIF: Analyze (analysis) OBJ: Conduct a sleep history for a patient.
TOP: Assessment MSC: Reduction of Risk Potential
MULTIPLE RESPONSE
1. The nurse is caring for a patient who has not been able to sleep well while in the hospital, leading to a disrupted sleep-wake cycle.
Which assessment findings will the nurse monitor for in this patient? (Select all that apply.)
a. Changes in physiological function such as temperature
b. Decreased appetite and weight loss
c. Anxiety, irritability, and restlessness
d. Shortness of breath and chest pain
e. Nausea, vomiting, and diarrhea
f. Impaired judgment
ANS: A, B, C, F
The biological rhythm of sleep frequently becomes synchronized with other body functions. Changes in body temperature correlate
with sleep pattern. When the sleep-wake cycle becomes disrupted, changes in physiological function such as temperature can
occur. Patients can experience decreased appetite, loss of weight, anxiety, restlessness, irritability, and impaired judgment.
Gastrointestinal and respiratory/cardiovascular symptoms such as shortness of breath and chest pain are not symptoms of a
disrupted sleep cycle.
DIF: Understand (comprehension)
OBJ: Explain the effect that the 24-hour sleep-wake cycle has on biological function.
TOP: Assessment MSC: Physiological Adaptation
Copyright © 2021, Elsevier Inc. All rights reserved. 7
2. The nurse is caring for a patient in the intensive care unit who is having trouble sleeping. The nurse explains the purpose of sleep
and its benefits. Which information will the nurse include in the teaching session? (Select all that apply.)
a. NREM sleep contributes to body tissue restoration.
b. During NREM sleep, biological functions increase.
c. Restful sleep preserves cardiac function.
d. Sleep contributes to cognitive restoration.
e. REM sleep decreases cortical activity.
ANS: A, C, D
Sleep contributes to physiological and psychological restoration. NREM sleep contributes to body tissue restoration. It allows the
body to rest and conserve energy. This benefits the cardiac system by allowing the heart to beat fewer times each minute. During
stage 4, the body releases growth hormone for renewal and repair of specialized cells such as the brain. During NREM sleep,
biological functions slow. REM sleep is necessary for brain tissue restoration and cognitive restoration and is associated with a
change in cerebral blood flow and increased cortical activity.
DIF: Understand (comprehension) OBJ: Explain the functions of sleep.
TOP: Teaching/Learning MSC: Physiological Adaptation
3. The patient and the nurse discuss the need for sleep. After the discussion, the patient is able to state factors that hinder sleep. Which
statements indicate the patient has a good understanding of the teaching? (Select all that apply.)
a. “Drinking coffee at 7 PM could interrupt my sleep.”
b. “Staying up late for a party can interrupt sleep patterns.”
c. “Exercising 2 hours before bedtime can decrease relaxation.”
d. “Changing the time of day that I eat dinner can disrupt sleep.”
e. “Worrying about work can disrupt my sleep.”
f. “Taking an antacid can decrease sleep.”
ANS: A, B, D, E
Caffeine, alcohol, and nicotine consumed late in the evening produce insomnia. Worry over personal problems or situations
frequently disrupts sleep. Alterations in routines, including changing mealtimes and staying up late at night for social activities, can
disrupt sleep. Exercising 2 hours before bedtime actually increases a sense of fatigue and promotes relaxation. Taking an antacid
does not decrease sleep.
DIF: Apply (application)
OBJ: Identify factors that normally promote and disrupt sleep. TOP: Teaching/Learning
MSC: Basic Care and Comfort
4. A community health nurse is providing an educational session at the senior center on how to promote sleep. Which practices should
the nurse recommend? (Select all that apply.)
a. Take a nap in the afternoon.
b. Sleep where you sleep best.
c. Use sedatives as a last resort.
d. Watch television right before sleep.
e. Decrease fluids 2 to 4 hours before sleep.
f. Get up if unable to fall asleep in 20 minutes.
ANS: B, C, E, F
The nurse should instruct the patient to sleep where he or she sleeps best, to use sedatives as a last resort, to decrease fluid intake to
cut down on bathroom trips, and, if unable to sleep in 20 minutes, to get up out of bed. Naps should be eliminated if they are not
part of the individual’s routine schedule, and if naps are taken, they should be limited to 20 minutes or less a day. Television can
stimulate and disrupt sleep patterns.
DIF: Apply (application)
OBJ: Identify nursing interventions designed to promote normal sleep cycles for patients of all ages.
TOP: Teaching/Learning MSC: Health Promotion and Maintenance
MATCHING
The nurse is caring for a group of patients who have sleeping disruptions. Match the condition to the intervention the nurse will
use.
a. Use continuous positive airway pressure.
b. Offer a small meal several hours before bedtime.
c. Administer antidepressants.
d. Administer modafinil
e. Do not startle.
f. Administer benzodiazepine-like drugs.
1. Cataplexy
2. Narcolepsy
3. Insomnia
4. Hiatal hernia
5. Sleepwalking
6. Obstructive sleep apnea
1. ANS: C DIF: Understand (comprehension)
OBJ: Identify nursing interventions designed to promote normal sleep cycles for patients of all ages.
TOP: Implementation MSC: Physiological Adaptation
2. ANS: D DIF: Understand (comprehension)
OBJ: Identify nursing interventions designed to promote normal sleep cycles for patients of all ages.
TOP: Implementation MSC: Physiological Adaptation
Copyright © 2021, Elsevier Inc. All rights reserved. 8
3. ANS: F DIF: Understand (comprehension)
OBJ: Identify nursing interventions designed to promote normal sleep cycles for patients of all ages.
TOP: Implementation MSC: Physiological Adaptation
4. ANS: B DIF: Understand (comprehension)
OBJ: Identify nursing interventions designed to promote normal sleep cycles for patients of all ages.
TOP: Implementation MSC: Physiological Adaptation
5. ANS: E DIF: Understand (comprehension)
OBJ: Identify nursing interventions designed to promote normal sleep cycles for patients of all ages.
TOP: Implementation MSC: Physiological Adaptation
6. ANS: A DIF: Understand (comprehension)
OBJ: Identify nursing interventions designed to promote normal sleep cycles for patients of all ages.
TOP: Implementation MSC: Physiological Adaptation

TB Q&A 10th 42 Fluid, Electrolyte, and Acid-Base Balance

Chapter 42: Fluid, Electrolyte, and Acid-Base Balance

MULTIPLE CHOICE

1. A patient is experiencing dehydration. While planning care, the nurse considers that the majority of the patient’s total water volume exists in with compartment?

a. Intracellular
b. Extracellular
c. Intravascular
d. Transcellular

 

ANS: A Intracellular
Intracellular (inside the cells) fluid accounts for approximately two-thirds of total body water. Extracellular (outside the cells) is approximately one-third of the total body water. Intravascular fluid (liquid portion of the blood) and transcellular fluid are two major divisions of the extracellular compartment.

 

2. The nurse is teaching about the process of passively moving water from an area of lower particle concentration to an area of higher particle concentration. Which process is the nurse describing?

a. Osmosis
b. Filtration
c. Diffusion
d. Active transport

 

ANS: A
The process of moving water from an area of low particle concentration to an area of higher particle concentration is known as osmosis. Filtration is mediated by fluid pressure from an area of higher pressure to an area of lower pressure. Diffusion is the passive movement of electrolytes or other particles down the concentration gradient (from areas of higher concentration to areas of lower concentration). Active transport requires energy in the form of adenosine triphosphate (ATP) to move electrolytes across cell membranes against the concentration gradient (from areas of lower concentration to areas of higher concentration).

 

3. The nurse observes edema in a patient who is experiencing venous congestion as a result of right heart failure. Which type of did pressure facilitate the formation of the patient’s edema?

a. Osmotic
b. Oncotic
c. Hydrostatic
d. Concentration

 

ANS: C Hydrostatic
Venous congestion increases capillary hydrostatic pressure. Increased hydrostatic pressure causes edema by causing increased movement of fluid into the interstitial area. Osmotic and oncotic pressures involve the concentrations of solutes and can contribute to edema in other situations, such as inflammation or malnutrition. Concentration pressure is not a nursing term.

 

 

4. The nurse administers an intravenous (IV) hypertonic solution to a patient expects the fluid shift to occur in what direction?

a. From intracellular to extracellular
b. From extracellular to intracellular
c. From intravascular to intracellular
d. From intravascular to interstitial

 

 

ANS: A
Hypertonic solutions will move fluid from the intracellular to the extracellular (intravascular). A hypertonic solution has a concentration greater than normal body fluids, so water will shift out of cells because of the osmotic pull of the extra particles. Movement of water from the extracellular (intravascular) into cells (intracellular) occurs when hypotonic fluids are administered. The distribution of fluid between intravascular and interstitial spaces occurs by filtration, the net sum of hydrostatic and osmotic pressures.

 

 

5. A nurse is preparing to start peripheral intravenous (IV) therapy. In which order will the nurse perform the steps starting with the first one?

1. Clean site.
2. Select vein.
3. Apply tourniquet.
4. Release tourniquet.
5. Reapply tourniquet.
6. Advance and secure.
7. Insert vascular access device.
a. 1, 3, 2, 7, 5, 4, 6
b. 1, 3, 2, 5, 7, 6, 4
c. 3, 2, 1, 5, 7, 6, 4
d. 3, 2, 4, 1, 5, 7, 6

 

 

ANS: D  3, 2, 4, 1, 5, 7, 6
The steps for inserting an intravenous catheter are as follows: Apply tourniquet, select vein, release tourniquet, clean site, reapply tourniquet, insert vascular access device, and advance and secure.

 

 

6. The nurse is laboratory blood results will expect to observe which cation in the most abundance?

a. Sodium
b. Chloride
c. Potassium
d. Magnesium

 

 

ANS: A: Sodium
Sodium is the most abundant cation in the blood. Potassium is the predominant intracellular cation. Chloride is an anion (negatively charged) rather than a cation (positively charged). Magnesium is found predominantly inside cells and in bone.

 

 

7. The nurse receives the patient’s most recent blood work results. Which laboratory value is of greatest concern?

a. Sodium of 145 mEq/L
b. Calcium of 15.5 mg/dL
c. Potassium of 3.5 mEq/L
d. Chloride of 100 mEq/L

 

 

ANS: B Calcium of 15.5 mg/dL
The normal calcium range is 9 to 10.5 mg/dL; therefore, a value of 15.5 mg/dL is abnormally high and of concern. The rest of the laboratory values are within their normal ranges: sodium 136 to 145 mEq/L, potassium 3.5 to 5.0 mEq/L, and chloride 98 to 106 mEq/L.

 

 

8. The nurse observes that the patient’s calcium is elevated. When checking the phosphate level, what does the nurse expect to see?

a. An increase
b. A decrease
c. Equal to calcium
d. No change in phosphate

 

 

ANS: B: A decrease
Phosphate will decrease. Serum calcium and phosphate have an inverse relationship. When one is elevated, the other decreases, except in some patients with end-stage renal disease.

 

 

9. Four patients arrive at the emergency department at the same time. Which patient will the nurse see first?

a. An infant with a temperature of 102.2° F and diarrhea for 3 days
b. A teenager with a sprained ankle and excessive edema
c. A middle-aged adult with abdominal pain who is moaning and holding her
stomach
d. An older adult with nausea and vomiting for 3 days with blood pressure 112/60

 

 

ANS: A: An infant with a temperature of 102.2° F and diarrhea for 3 days.
The infant should be seen first. An infant’s proportion of total body water (70% to 80% total body weight) is greater than that of children or adults. Infants and young children have greater water needs and immature kidneys. They are at greater risk for extracellular volume deficit and hypernatremia because body water loss is proportionately greater per kilogram of weight. A teenager with excessive edema from a sprained ankle can wait. A middle-aged adult moaning in pain can wait as can an older adult with a blood pressure of 112/60.

 

 

10. The patient has an intravenous (IV) line and the nurse needs to remove the gown. In which order will the nurse perform the steps, starting with the first one?

1. Remove the sleeve of the gown from the arm without the IV.
2. Remove the sleeve of the gown from the arm with the IV.
3. Remove the IV solution container from its stand.
4. Pass the IV bag and tubing through the sleeve.
a. 1, 2, 3, 4
b. 2, 3, 4, 1
c. 3, 4, 1, 2
d. 4, 1, 2, 3

 

 

ANS: A: 1, 2, 3, 4
Change regular gowns by following these steps for maximum speed and arm mobility: (1) To remove a gown, remove the sleeve of the gown from the arm without the IV line, maintaining the patient’s privacy. (2) Remove the sleeve of the gown from the arm with the IV line. (3) Remove the IV solution container from its stand and pass it and the tubing through the sleeve. (If this involves
removing the tubing from an EID, use the roller clamp to slow the infusion to prevent the accidental infusion of a large volume of solution or medication.)

 

 

11. A 2-year-old child has ingested a quantity of medication that causes respiratory depression. For which acid-base imbalance will the nurse most closely monitors this child?

a. Respiratory alkalosis
b. Respiratory acidosis
c. Metabolic acidosis
d. Metabolic alkalosis

 

 

ANS: B: Respiratory acidosis
Respiratory depression leads to hypoventilation. Hypoventilation results in retention of CO2 and respiratory acidosis. Respiratory alkalosis would result from hyperventilation, causing a decrease in CO2 levels. The metabolic acid-base imbalance would be a result of kidney dysfunction, vomiting, diarrhea, or other conditions that affect metabolic acids.

 

 

12. A patient is admitted for a bowel obstruction and has had a nasogastric tube set to low intermittent suction for the past 3 days. Which arterial blood gas values will the nurse expect to observe?

a. Respiratory alkalosis
b. Metabolic alkalosis
c. Metabolic acidosis
d. Respiratory acidosis

 

 

ANS: B Metabolic alkalosis
The patient is losing acid from the nasogastric tube so the patient will have metabolic alkalosis. Lung problems will produce respiratory alkalosis or acidosis. Metabolic acidosis will occur when too much acid is in the body like kidney failure.

 

 

13. Which blood gas result will the nurse expect to observe in a patient with respiratory alkalosis?

a. pH 7.60, PaCO2 40 mm Hg, HCO3– 30 mEq/L
b. pH 7.53, PaCO2 30 mm Hg, HCO3– 24 mEq/L
c. pH 7.35, PaCO2 35 mm Hg, HCO3– 26 mEq/L
d. pH 7.25, PaCO2 48 mm Hg, HCO3– 23 mEq/L

 

 

ANS: B
Respiratory alkalosis should show an alkalotic pH and decreased CO2  (respiratory) values, with a normal HCO3 –. In this case, pH 7.53 is alkaline (normal = 7.35 to 7.45), PaCO2 is 30 (normal 35 to 45 mm Hg), and HCO3
– is 24 (normal = 22 to 26 mEq/L). A result of pH 7.60, PaCO2 40 mm Hg, HCO3 – 30 mEq/L is metabolic alkalosis. pH 7.35, PaCO2 35 mm Hg, HCO3
– 26 mEq/L is within normal limits. pH 7.25, PaCO2 48 mm Hg, HCO3
– 23 mEq/L is respiratory acidosis.

 

 

14. A nurse is caring for a patient whose electrocardiogram (ECG) presents with changes characteristic of hypokalemia. Which assessment finding will the nurse expect?

a. Dry mucous membranes
b. Abdominal distention
c. Distended neck veins
d. Flushed skin

 

 

ANS: B Abdominal distention
Signs and symptoms of hypokalemia are muscle weakness, abdominal distention, decreased bowel sounds, and cardiac dysrhythmias. Distended neck veins occur in fluid overload. Thready peripheral pulses indicate hypovolemia. Dry mucous membranes and flushed skin are indicative of dehydration and hypernatremia.

 

 

15. In which patient will the nurse expect to see a positive Chvostek’s sign?

a. A 7-year-old child admitted for severe burns
b. A 24-year-old adult admitted for chronic alcohol abuse
c. A 50-year-old patient was admitted for an acute exacerbation of hyperparathyroidism
d. A 75-year-old patient was admitted for a broken hip related to osteoporosis

 

 

ANS: B A 24-year-old adult admitted for chronic alcohol abuse                                    A positive Chvostek’s sign is representative of hypocalcemia or hypomagnesemia. Hypomagnesemia is common with alcohol abuse. Hypocalcemia can be brought on by alcohol abuse and pancreatitis (which also can be affected by alcohol consumption). Burn patients frequently experience extracellular fluid volume deficits. Hyperparathyroidism causes hypercalcemia. Immobility is associated with hypercalcemia.

 

 

16. A patient is experiencing respiratory acidosis. Which organ system is responsible for compensation in this patient?

a. Renal
b. Endocrine
c. Respiratory
d. Gastrointestinal

 

 

ANS: A Renal
The kidneys (renal) are responsible for respiratory acidosis compensation. A problem with the respiratory system causes respiratory acidosis, so another organ system (renal) needs to compensate. Problems with the gastrointestinal and endocrine systems can cause acid-base imbalances, but these systems cannot compensate for an existing imbalance.

 

 

17. A nurse is caring for a patient prescribed peripheral intravenous (IV) therapy. Which task will the nurse assign to the nursing assistive personnel?

a. Recording intake and output
b. Regulating intravenous flow rate
c. Starting peripheral intravenous therapy
d. Changing a peripheral intravenous dressing

 

 

ANS: A Recording intake and output
A nursing assistive personnel (AP) can record intake and output. An RN cannot delegate regulating flow rate, starting an IV, or changing an IV dressing to a NAP.

 

 

18. The nurse is caring for a diabetic patient with renal failure who is in metabolic acidosis. Which laboratory findings are consistent with metabolic acidosis?

a. pH 7.3, PaCO2 36 mm Hg, HCO3– 19 mEq/L
b. pH 7.5, PaCO2 35 mm Hg, HCO3– 35 mEq/L
c. pH 7.32, PaCO2 47 mm Hg, HCO3– 23 mEq/L
d. pH 7.35, PaCO2 40 mm Hg, HCO3– 25 mEq/L

 

 

ANS: A  pH 7.3, PaCO2 36 mm Hg, HCO3– 19 mEq/L
The laboratory values that reflect metabolic acidosis are pH 7.3, PaCO2 36 mm Hg, HCO3 – 19 mEq/L. A laboratory finding of pH 7.5, PaCO2 35 mm Hg, HCO3– 35 mEq/L is metabolic alkalosis. pH 7.32, PaCO2 47 mm Hg, HCO3– 23 mEq/L is respiratory acidosis. pH 7.35, PaCO2 40 mm Hg, HCO3 – 25 mEq/L values are within the normal range.

 

 

19. The nurse is assessing a patient and notes crackles in the lung bases and neck vein distention. Which action will the nurse take first?

a. Offer calcium-rich foods.
b. Administer diuretic.
c. Raise the head of the bed.
d. Increase fluids.

 

 

ANS: C
The patient is in fluid overload. Raising the head of the bed to a high Fowler’s position will help ease breathing and so is the first action. Offering calcium-rich foods is for hypocalcemia, not fluid overload. Administering a diuretic is the second action. Increasing fluids is contraindicated and would make the situation worse.

 

 

20. A patient receiving chemotherapy has gained 5 pounds in 2 days. Which assessment question by the nurse is most appropriate?

a. “Are you following any weight loss program?”
b. “How many calories a day do you consume?”
c. “Do you have a dry mouth or feel thirsty?”
d. “How many times a day do you urinate?”

 

 

ANS: D  “How many times a day do you urinate?”
A rapid gain in weight usually indicates extracellular volume (ECV) excess if the person began with normal ECV. Asking the patient about urination habits will help determine whether the body is trying to excrete the excess fluid or if renal dysfunction is contributing to ECV excess. This is too rapid a weight gain to be dietary; it is fluid retention. Asking about following a weight loss program will not help determine the cause of the problem. Caloric intake does not account for rapid weight changes. Dry mouth and thirst accompany ECV deficit, which would be associated with rapid weight loss.

 

 

21. The health care provider has ordered a hypotonic intravenous (IV) solution to be administered. Which IV bag will the nurse prepare?

a. 0.45% sodium chloride (1/2 NS)
b. 0.9% sodium chloride (NS)
c. Lactated Ringer’s (LR)
d. Dextrose 5% in Lactated Ringer’s (D5LR)

 

 

ANS: A 0.45% sodium chloride (1/2 NS)
0.45% sodium chloride is a hypotonic solution. NS and LR are isotonic. D5LR is hypertonic.

 

 

22. The health care provider asks the nurse to monitor the fluid volume status of a heart failure patient and a patient at risk for clinical dehydration. Which is the most effective nursing intervention for monitoring both of these patients?

a. Assess the patients for edema in extremities.
b. Ask the patients to record their intake and output.
c. Weigh the patients every morning before breakfast.
d. Measure the patients’ blood pressures every 4 hours.

 

 

ANS: C Weigh the patients every morning before breakfast.
An effective measure of fluid retention or loss is daily weights; each kg (2.2 pounds) change is equivalent to 1 L of fluid gained or lost. This measurement should be performed at the same time every day using the same scale and the same amount of clothing. Although intake and output records are important assessment measures, some patients are not able to keep their own records
themselves. Blood pressure can decrease with extracellular volume (ECV) deficit but will not necessarily increase with recent ECV excess (heart failure patient). Edema occurs with ECV excess but not with clinical dehydration.

 

 

23. A nurse is caring for a patient diagnosed with cancer who presents with anorexia, blood pressure 100/60, and elevated white blood cell count. Which primary purpose for starting total parenteral nutrition (TPN) will the nurse add to the care plan?

a. Stimulate the patient’s appetite to eat.
b. Deliver antibiotics to fight off infection.
c. Replace fluid, electrolytes, and nutrients.
d. Provide medication to raise blood pressure.

 

 

ANS: C Replace fluid, electrolytes, and nutrients.
Total parenteral nutrition is an intravenous solution composed of nutrients and electrolytes to replace the ones the patient is not eating or losing. TPN does not stimulate the appetite. TPN does not contain blood pressure medication or antibiotics.

 

 

24. A patient presents to the emergency department with reports of vomiting and diarrhea for the past 48 hours. The health care provider orders isotonic intravenous (IV) therapy. Which IV will the nurse prepare?

a. 0.225% sodium chloride (1/4 NS)
b. 0.45% sodium chloride (1/2 NS)
c. 0.9% sodium chloride (NS)
d. 3% sodium chloride (3% NaCl)

 

 

ANS: C
Patients with prolonged vomiting and diarrhea become hypovolemic. A solution to replace extracellular volume is 0.9% sodium chloride, which is an isotonic solution. 0.225% and 0.45% sodium chloride are hypotonic. 3% sodium chloride is hypertonic.

 

 

25. A nurse administering a diuretic to a patient is teaching about foods to increase in the diet. Which food choices by the patient will best indicate successful teaching?

a. Milk and cheese
b. Potatoes and fresh fruit
c. Canned soups and vegetable
d. Whole grains and dark green leafy vegetables

 

 

ANS: B  Potatoes and fresh fruit
Potatoes and fruits are high in potassium. Milk and cheese are high in calcium. Canned soups and vegetables are high in sodium. Whole grains and dark green leafy vegetables are high in magnesium.

 

26. The nurse is evaluating the effectiveness of intravenous fluid therapy in a patient with hypernatremia. Which finding indicates goal achievement?

a. Urine output increases to 150 mL/hr.
b. Systolic and diastolic blood pressure decreases.
c. Serum sodium concentration returns to normal.
d. Large amounts of emesis and diarrhea decrease.

 

 

ANS: C: Serum sodium concentration returns to normal.
Hypernatremia is diagnosed by elevated serum sodium concentration. Blood pressure is not an accurate indicator of hypernatremia.  Emesis and diarrhea will not stop because of intravenous therapy. Urine output is influenced by many factors, including extracellular fluid volume. Large dilute urine output can cause further hypernatremia.

 

 

27. The nurse is calculating intake and output on a patient. The patient drinks 150 mL of orange juice at breakfast, voids 125 mL after breakfast, vomits 250 mL of greenish fluid, sucks on 60 mL of ice chips, and for lunch consumes 75 mL of chicken broth. Which totals for intake and output will the nurse document in the patient’s medical record?

a. Intake 255; output 375
b. Intake 285; output 375
c. Intake 505; output 125
d. Intake 535; output 125

 

 

ANS: A: Intake 255; output 375
Intake = 150 mL of orange juice, 60 mL of ice chips (but only counts as 30 since ice chips are half of the amount), and 75 mL of chicken broth; 150 + 30 + 75 = 255. Output = 125 mL of urine (void) and 250 mL of vomitus; 125 + 250 = 375.

 

 

28. Which assessment finding should cause a nurse to further assess for extracellular fluid volume deficit?

a. Moist mucous membranes
b. Postural hypotension
c. Supple skin turgor
d. Pitting edema

 

 

ANS: B Postural hypotension
Physical examination findings of deficit include postural hypotension, tachycardia, thready pulse, dry mucous membranes, and poor skin turgor. Pitting edema indicates that the patient may be retaining excess extracellular fluid.

 

 

29. A patient is to receive 1000 mL of 0.9% sodium chloride intravenously at a rate of 125 mL/hr. The nurse is using micro drip gravity drip tubing. Which rate will the nurse calculate for the minute flow rate (drops/min)?

a. 12 drops/min
b. 24 drops/min
c. 125 drops/min
d. 150 drops/min

 

 

ANS: C: 125 drops/min
Microdrip tubing delivers 60 drops/mL. Calculation for a rate of 125 mL/hr using microdrip tubing: (125 mL/1 hr)(60 drops/1mL)(1 hr/60 min) = 125 drop/min.

 

 

30. A nurse begins infusing a 250-mL bag of IV fluid at 1845 on Monday and programs the pump to infuse at 50 mL/hr. At what time should the infusion be completed?

a. 2300 Monday
b. 2345 Monday
c. 0015 Tuesday
d. 0045 Tuesday

 

 

ANS: B: 2345 Monday
250 mL ÷ 50 mL/hr = 5 hr 1845 + 5 hr = 2345, which would be 2345 on Monday.

 

31. A nurse caring for a diabetic patient with a bowel obstruction has orders to ensure that the volume of intake matches the output. In the past 4 hours, the patient received dextrose 5% with 0.9% sodium chloride through a 22-gauge catheter infusing at 150 mL/hr and has eaten 200 mL of ice chips. The patient also has an NG suction tube set to low continuous suction that had 300-mL output. The patient has voided 400 mL of urine. After reporting these values to the health care provider, which order does the nurse anticipate?

a. Add a potassium supplement to replace loss from the output.
b. Decrease the rate of intravenous fluids to 100 mL/hr.
c. Administer a diuretic to prevent fluid volume excess.
d. Discontinue the nasogastric suctioning.

 

ANS: A: Add a potassium supplement to replace loss from the output.
The total fluid intake and output equal 700 mL, which meets the provider’s goals. Patients with nasogastric suctioning are at risk for potassium deficit, so the nurse would anticipate a potassium supplement to correct this condition. Remember to record half the volume of ice chips when calculating intake. The other measures would be unnecessary because the net fluid volume is equal.

 

 

32. A nurse is caring for a patient who is receiving peripheral intravenous (IV) therapy. When the nurse is flushing the patient’s peripheral IV, the patient reports pain. Upon assessment, the nurse notices a red streak that is warm to the touch. What is the nurse’s initial action?

a. Record a phlebitis grade of 4.
b. Assign an infiltration grade.
c. Apply moist compress.
d. Discontinue the IV.

 

 

ANS: D Discontinue the IV.
The IV site has phlebitis. The nurse should discontinue the IV. The phlebitis score is 3. The site has phlebitis, not infiltration. A moist compress may be needed after the IV is discontinued.

 

 

33. A nurse is assisting the health care provider in inserting a central line. Which action indicates the nurse is following the recommended bundle protocol to reduce central line-associated bloodstream infections (CLABSI)?

a. Preps skin with povidone-iodine solution.
b. Suggests the femoral vein for insertion site.
c. Applies double gloving without hand hygiene.
d. Uses chlorhexidine skin antisepsis prior to insertion.

 

 

ANS: D   Uses chlorhexidine skin antisepsis prior to insertion.                                    A recommended bundle at the insertion of a central line is hand hygiene prior to catheter insertion, use of maximum sterile barrier precautions upon insertion, chlorhexidine skin antisepsis prior to insertion and during dressing changes, avoidance of the femoral vein for central venous access for adults, and daily evaluation of line necessity, with prompt removal of nonessential lines. Povidone-iodine is not recommended.

 

34. The nurse is caring for a group of patients. Which patient will the nurse see first?

a. A patient with D5W hanging with the blood
b. A patient with type A blood receiving type O blood
c. A patient with intravenous potassium chloride that is diluted
d. A patient with a right mastectomy and an intravenous site in the left arm

 

ANS: A:  A patient with D5W hanging with the blood
The nurse will see the patient with D5W and blood to prevent a medication error. When preparing to administer blood, prime the tubing with 0.9% sodium chloride (normal saline) to prevent hemolysis or breakdown of RBCs. All the rest are normal. A patient with type A blood can receive type O. Type O is considered the universal donor. A patient with a mastectomy should have the IV in the other arm. Potassium chloride should be diluted, and it is never given IV push.

 

35. A nurse is administering a blood transfusion. Which assessment finding will the nurse report immediately?

a. Blood pressure 110/60
b. Temperature 101.3° F
c. Poor skin turgor and pallor
d. Heart rate of 100 beats/min

 

ANS: B: Temperature 101.3° F
A fever should be reported immediately, and the blood transfusion stopped. All other assessment findings are expected. Blood is given to elevate blood pressure, improve pallor, and decrease tachycardia.

 

 

36. A nurse has just received a bag of packed red blood cells (RBCs) for a patient. What is the longest time the nurse can let the blood infuse?

a. 30 minutes
b. 2 hours
c. 4 hours
d. 6 hours

 

 

 

ANS: C: 4 hours
Ideally, a unit of whole blood or packed RBCs is transfused in 2 hours. This time can be lengthened to 4 hours if the patient is at risk for extracellular volume excess. Beyond 4 hours there is a risk for bacterial contamination of the blood.

 

37. A patient has an acute intravascular hemolytic reaction to a blood transfusion. After discontinuing the blood transfusion, which is the nurse’s next action?

a. Discontinue the IV catheter.
b. Return the blood to the blood bank.
c. Run normal saline through the existing tubing.
d. Start normal saline at TKO rate using new tubing.

 

ANS: D: Start normal saline at TKO rate using new tubing.
The nurse should first attach new tubing and begin running in normal saline at a rate to keep the vein open, in case any medications need to be delivered through an IV site. The existing tubing should not be used because that would infuse the blood in the tubing into the patient. It is necessary to preserve the IV catheter in place for IV access to treat the patient. After the patient has been assessed and stabilized, the blood can be returned to the blood bank.

 

 

38. A nurse assessing a patient who is receiving a blood transfusion finds that the patient is anxiously fidgeting in bed. The patient is afebrile but dyspneic. The nurse auscultates crackles in both lung bases and sees jugular vein distention. On which transfusion complication will the nurse focus interventions?

a. Fluid volume excess
b. Hemolytic reaction
c. Anaphylactic shock
d. Septicemia

 

 

ANS: A Fluid volume excess
The signs and symptoms are concurrent with fluid volume excess. The anaphylactic shock would have presented with urticaria, dyspnea, and hypotension. Septicemia would include a fever. A hemolytic reaction would consist of flank pain, chills, and fever.

 

 

39. A nurse preparing to start a blood transfusion will use which type of tubing?

a. Two-way valves to allow the patient’s blood to mix and warm the blood
transfusing
b. An injection port to mix additional electrolytes into the blood
c. One with a filter to ensure that clots do not enter the patient
d. An air vent to let bubbles into the blood

 

 

ANS: C  One with a filter to ensure that clots do not enter the patient
When administering a transfusion, you need an appropriate-size IV catheter and blood administration tubing that has a special in-line filter. The patient’s blood should not be mixed with the infusion blood. Air bubbles should not be allowed to enter the blood. The only substance compatible with blood is normal saline; no additives should be mixed with the infusing blood.

 

 

40. The nurse is caring for a patient with hyperkalemia. Which body system assessment is the priority?

a. Gastrointestinal
b. Neurological
c. Respiratory
d. Cardiac

 

ANS: D Cardiac
Cardiac is the priority. Hyperkalemia places the patient at risk for potentially serious dysrhythmias and cardiac arrest. Potassium balance is necessary for cardiac function. Respiratory is the priority with hypokalemia. Monitoring of gastrointestinal and neurological systems would be indicated for other electrolyte imbalances.

 

 

41. Which assessment finding will the nurse expect for a patient with the following laboratory values: sodium 145 mEq/L, potassium 4.5 mEq/L, calcium 4.5 mg/dL?

a. Weak quadriceps muscles
b. Decreased deep tendon reflexes
c. Light-headedness when standing up
d. Tingling of extremities with possible tetany

 

 

ANS: D Tingling of extremities with possible tetany
This patient has hypocalcemia because the normal calcium range is 8.4 to 10.5 mg/dL. Hypocalcemia causes muscle tetany, positive Chvostek’s sign, and tingling of the extremities. Sodium and potassium values are within their normal ranges: sodium 135 to 145 mEq/L; potassium 3.5 to 5.0 mEq/L. Light-headedness when standing up is a manifestation of ECV deficit or sometimes
hypokalemia. Weak quadriceps muscles are associated with potassium imbalances. Decreased deep tendon reflexes are related to hypercalcemia or hypermagnesemia.

 

 

42. While the nurse is taking a patient history, the nurse discovers the patient has a type of diabetes that results from a head injury and does not require insulin. Which dietary change should the nurse share with the patient?

a. Reduce the quantity of carbohydrates ingested to lower blood sugar.
b. Include a serving of dairy in each meal to elevate calcium levels.
c. Drink plenty of fluids throughout the day to stay hydrated.
d. Avoid foods high in acid to avoid metabolic acidosis.

 

 

ANS: C Drink plenty of fluids throughout the day to stay hydrated.
The patient has diabetes insipidus, which places the patient at risk for dehydration and hypernatremia. Dehydration should be prevented by drinking plenty of fluids to replace the extra water excreted in the urine. Foods high in acid are not what cause metabolic acidosis. A reduction in carbohydrates to lower blood sugar will not help a patient with diabetes insipidus but it may help
a patient with diabetes mellitus. Calcium-rich dairy products would be recommended for a hypocalcemic patient.
DIF: Apply (application)

MULTIPLE RESPONSE

1. A nurse is selecting a site to insert an intravenous (IV) catheter on an adult. Which actions will the nurse take? (Select all that apply.)

a. Check for contraindications to the extremity.
b. Start proximally and move distally on the arm.
c. Choose a vein with minimal curvature.
d. Choose the patient’s dominant arm.
e. Select a vein that is rigid.
f. Avoid areas of flexion.

 

 

ANS: A, Check for contraindications to the extremity. C, Choose a vein with minimal curvature. F Avoid areas of flexion.
The vein should be relatively straight to avoid catheter occlusion. Contraindications to starting an IV catheter are conditions such as mastectomy, AV fistula, and central line in the extremity and should be checked before initiation of IV. Avoid areas of flexion if possible. The nurse should start distally and move proximally, choosing the nondominant arm if possible. The nurse should feel for the best location; a good vein should feel spongy; a rigid vein should be avoided because it might have had previous trauma or damage.

 

2. Which assessments will alert the nurse that a patient’s IV has infiltrated? (Select all that apply.)

a. Edema of the extremity near the insertion site
b. Reddish streak is proximal to the insertion site
c. Skin discolored or pale in appearance
d. Pain and warmth at the insertion site
e. Palpable venous cord
f. Skin cool to the touch

 

ANS: A, Edema of the extremity near the insertion site. C, Skin discolored or pale in appearance. F Skin cool to the touch.
Infiltration results in skin that is edematous near the IV insertion site. Skin is cool to the touch and maybe pale or discolored. Pain, warmth, erythema, a reddish streak, and a palpable venous cord are all symptoms of phlebitis.

3. A nurse is discontinuing a patient’s peripheral IV access. Which actions should the nurse take? (Select all that apply.)

a. Wear sterile gloves and a mask.
b. Stop the infusion before removing the IV catheter.
c. Use scissors to remove the IV site dressing and tape.
d. Apply firm pressure with sterile gauze during removal.
e. Keep the catheter parallel to the skin while removing it.
f. Apply pressure to the site for 2 to 3 minutes after removal.

 

 

ANS: B, top the infusion before removing the IV catheter. E, Keep the catheter parallel to the skin while removing it. F Apply pressure to the site for 2 to 3 minutes after removal.
The nurse should stop the infusion before removing the IV catheter, so the fluid does not drip on the patient’s skin; keep the catheter parallel to the skin while removing it to reduce trauma to the vein; and apply pressure to the site for 2 to 3 minutes after removal to decrease bleeding from the site. Scissors should not be used because they may accidentally cut the catheter or tubing or may injure the patient. During removal of the IV catheter, light pressure, not firm pressure, is indicated to prevent trauma. Clean gloves are used for discontinuing peripheral IV access because gloved hands will handle the external dressing, tubing, and tape, which are not sterile.

COMPLETION

1. A patient has 250 mL of a jejunostomy feeding with 30 mL of water before and after feeding and 200 mL of urine. Thirty minutes later the patient has 100 mL of diarrhea. At 1300 the patient receives 150 mL of blood and voids another 200 mL. Calculate the patient’s intake. Record your answer as a whole number. _____ mL

 

ANS:460
The patient’s fluid intake is 250 mL of feeding, 60 mL of water (30 mL before and after), and 150 blood: 250 + 60 + 150 = 460 mL. Fluid intake includes all liquids that a person eats (e.g., gelatin, ice cream, soup), drinks (e.g., water, coffee, juice), or receives through nasogastric or jejunostomy feeding tubes. IV fluids (continuous infusions and intermittent IV piggybacks) and blood
components also are sources of intake. Fluid output includes urine, diarrhea, vomitus, gastric suction, and drainage from postsurgical wounds or other tubes.

MATCHING

A nurse is monitoring patients for fluid and electrolyte and acid-base imbalances. Match the body’s regulators to the function it provides.

a. Increases excretion of sodium and water.
b. Reduces excretion of sodium and water.
c. Reduces excretion of water.
d. Major buffer in the extracellular fluid.
e. Vasoconstricts and stimulates aldosterone release.
1. Antidiuretic hormone
2. Angiotensin II
3. Aldosterone
4. Atrial natriuretic peptide
5. Bicarbonate
1. ANS: C Reduces excretion of water.
2. ANS: E Vasoconstricts and stimulates aldosterone release.                                    3. ANS: B Reduces excretion of sodium and water.
4. ANS: A Increases excretion of sodium and water.
5. ANS: D Bicarbonate

TB Q&A 10th 41 OXYGENATION

Chapter 41: Oxygenation

MULTIPLE CHOICE

1. A nurse is teaching staff about the conduction of the heart. In which order will the nurse present the conduction cycle, starting with
the first structure?
1. Bundle of His
2. Purkinje network
3. Intraatrial pathways
4. Sinoatrial (SA) node
5. Atrioventricular (AV) node
a. 5, 4, 3, 2, 1
b. 4, 3, 5, 1, 2
c. 4, 5, 3, 1, 2
d. 5, 3, 4, 2, 1
ANS: B
The conduction system originates with the SA node, the “pacemaker” of the heart. The electrical impulses are transmitted through
the atria along intraatrial pathways to the AV node. It assists atrial emptying by delaying the impulse before transmitting it through
the Bundle of His and the ventricular Purkinje network.
DIF: Understand (comprehension)
OBJ: Describe the structure and function of the cardiopulmonary system.
TOP: Teaching/Learning MSC: Physiological Adaptation
2. A nurse is teaching the patient with mitral valve problems about the valves in the heart. Starting on the right side of the heart,
describe the sequence of the blood flow through these valves.
1. Mitral
2. Aortic
3. Tricuspid
4. Pulmonic
a. 1, 3, 2, 4
b. 4, 3, 2, 1
c. 3, 4, 1, 2
d. 2, 4, 1, 3
ANS: C
The blood flows through the valves in the following direction: tricuspid, pulmonic, mitral, and aortic.
DIF: Understand (comprehension)
OBJ: Describe the structure and function of the cardiopulmonary system.
TOP: Teaching/Learning MSC: Physiological Adaptation
3. A nurse explains the function of the alveoli to a patient with respiratory problems. Which information about the alveoli’s function
will the nurse share with the patient?
a. Carries out gas exchange.
b. Regulates tidal volume.
c. Produces hemoglobin.
d. Stores oxygen.
ANS: A
The alveolus is a capillary membrane that allows gas exchange of oxygen and carbon dioxide during respiration. The alveoli do not
store oxygen, regulate tidal volume, or produce hemoglobin.
DIF: Understand (comprehension)
OBJ: Describe the physiological processes of ventilation, perfusion, and exchange of respiratory gases. TOP: Teaching/Learning
MSC: Physiological Adaptation
4. A nurse auscultates heart sounds. When the nurse hears S2, which valves is the nurse hearing close?
a. Aortic and mitral
b. Mitral and tricuspid
c. Aortic and pulmonic
d. Mitral and pulmonic
ANS: C
As the ventricles empty, the ventricular pressures decrease, allowing closure of the aortic and pulmonic valves, producing the
second heart sound, S2. The mitral and tricuspid produce the first heart sound, S1. The aortic and mitral do not close at the same
time. The mitral and pulmonic do not close at the same time.
DIF: Apply (application)
OBJ: Describe the structure and function of the cardiopulmonary system.
TOP: Assessment MSC: Health Promotion and Maintenance
Copyright © 2021, Elsevier Inc. All rights reserved. 2
5. The nurse is teaching about the process of exchanging gases through the alveolar capillary membrane. Which term will the nurse
use to describe this process?
a. Ventilation
b. Surfactant
c. Perfusion
d. Diffusion
ANS: D
Diffusion is the process of gases exchanging across the alveoli and capillaries of body tissues. Ventilation is the process of moving
gases into and out of the lungs. Surfactant is a chemical produced in the lungs to maintain the surface tension of the alveoli and
keep them from collapsing. Perfusion is the ability of the cardiovascular system to carry oxygenated blood to tissues and return
deoxygenated blood to the heart.
DIF: Understand (comprehension)
OBJ: Describe the physiological processes of ventilation, perfusion, and exchange of respiratory gases. TOP: Teaching/Learning
MSC: Physiological Adaptation
6. A nurse is caring for a patient who was in a motor vehicle accident that resulted in cervical trauma to C4. Which assessment is the
priority?
a. Pulse
b. Respirations
c. Temperature
d. Blood pressure
ANS: B
Respirations and oxygen saturation are the priorities. Cervical trauma at C3 to C5 usually results in paralysis of the phrenic nerve.
When the phrenic nerve is damaged, the diaphragm does not descend properly, thus reducing inspiratory lung volumes and causing
hypoxemia. While pulse and blood pressure are important, respirations are the priority. Temperature is not a high priority in this
situation.
DIF: Analyze (analysis)
OBJ: Assess for the risk factors affecting a patient’s oxygenation.
TOP: Assessment MSC: Management of Care
7. The patient is breathing normally. Which process does the nurse consider is working properly when the patient inspires?
a. Stimulation of chemical receptors in the aorta
b. Reduction of arterial oxygen saturation levels
c. Requirement of elastic recoil lung properties
d. Enhancement of accessory muscle usage
ANS: A
Inspiration is an active process, stimulated by chemical receptors in the aorta. Reduced arterial oxygen saturation levels indicate
hypoxemia, an abnormal finding. Expiration is a passive process that depends on the elastic recoil properties of the lungs, requiring
little or no muscle work. Prolonged use of the accessory muscles does not promote effective ventilation and causes fatigue.
DIF: Understand (comprehension)
OBJ: State the process of the neural and chemical regulation of respiration.
TOP: Assessment MSC: Physiological Adaptation
8. The home health nurse recommends that a patient with respiratory problems install a carbon monoxide detector in the home. What
is the rationale for the nurse’s action?
a. Carbon monoxide detectors are required by law in the home.
b. Carbon monoxide tightly binds to hemoglobin, causing hypoxia.
c. Carbon monoxide signals the cerebral cortex to cease ventilations.
d. Carbon monoxide combines with oxygen in the body and produces a deadly toxin.
ANS: B
Carbon monoxide binds tightly to hemoglobin; therefore, oxygen is not able to bind to hemoglobin and be transported to tissues,
causing hypoxia. A carbon monoxide detector is not required by law, does not signal the cerebral cortex to cease ventilations, and
does not combine with oxygen but with hemoglobin to produce a toxin.
DIF: Understand (comprehension)
OBJ: Describe nursing care interventions used to promote oxygenation in the primary care, acute care, and restorative and continuing care
settings. TOP: Planning
MSC: Health Promotion and Maintenance
Copyright © 2021, Elsevier Inc. All rights reserved. 3
9. While performing an assessment, the nurse hears crackles in the patient’s lung fields. The nurse also learns that the patient is
sleeping on three pillows to help with the difficulty breathing during the night. Which condition will the nurse most likely observe
written in the patient’s medical record?
a. Atrial fibrillation
b. Myocardial ischemia
c. Left-sided heart failure
d. Right-sided heart failure
ANS: C
Left-sided heart failure results in pulmonary congestion, the signs and symptoms of which include shortness of breath, cough,
crackles, and paroxysmal nocturnal dyspnea (difficulty breathing when lying flat). Right-sided heart failure is systemic and results
in peripheral edema, weight gain, and distended neck veins. Atrial fibrillation is often described as an irregularly irregular rhythm;
rhythm is irregular because of the multiple pacemaker sites. Myocardial ischemia results when the supply of blood to the
myocardium from the coronary arteries is insufficient to meet myocardial oxygen demands, producing angina or myocardial
infarction.
DIF: Apply (application)
OBJ: Assess for the physical manifestations that occur with alterations in oxygenation.
TOP: Assessment MSC: Physiological Adaptation
10. A patient has experienced a myocardial infarction. On which primary blood vessel will the nurse focus care to reduce ischemia?
a. Superior vena cava
b. Pulmonary artery
c. Coronary artery
d. Carotid artery
ANS: C
A myocardial infarction is the lack of blood flow due to obstruction to the coronary artery, which supplies the heart with blood. The
superior vena cava returns blood back to the heart. The pulmonary artery supplies deoxygenated blood to the lungs. The carotid
artery supplies blood to the brain.
DIF: Understand (comprehension)
OBJ: Differentiate among the physiological processes of cardiac output, myocardial blood flow, and coronary artery circulation.
TOP: Planning MSC: Physiological Adaptation
11. A nurse is teaching a health class about the heart. Which information from the class members indicates teaching by the nurse is
successful for the flow of blood through the heart, starting in the right atrium?
a. Right ventricle, left ventricle, left atrium
b. Left atrium, right ventricle, left ventricle
c. Right ventricle, left atrium, left ventricle
d. Left atrium, left ventricle, right ventricle
ANS: C
Unoxygenated blood flows through the venae cavae into the right atrium, where it is pumped down to the right ventricle; the blood
is then pumped out the pulmonary artery and is returned oxygenated via the pulmonary vein to the left atrium, where it flows to the
left ventricle and is pumped out to the rest of the body via the aorta.
DIF: Understand (comprehension)
OBJ: Describe the structure and function of the cardiopulmonary system.
TOP: Teaching/Learning MSC: Health Promotion and Maintenance
12. The nurse suspects the patient has increased cardiac afterload. Which piece of equipment should the nurse obtain to determine the
presence of this condition?
a. Pulse oximeter
b. Oxygen cannula
c. Blood pressure cuff
d. Yankauer suction tip catheter
ANS: C
A blood pressure cuff is needed. The diastolic aortic pressure is a good clinical measure of afterload. Afterload is the resistance to
left ventricular ejection. In hypertension the afterload increases, making cardiac workload also increase. A pulse oximeter is used to
monitor the level of arterial oxygen saturation; it will not help determine increased afterload. While an oxygen cannula may be
needed to help decrease the effects of increased afterload, it will not help determine the presence of afterload. A Yankauer suction
tip catheter is used to suction the oral cavity.
DIF: Analyze (analysis)
OBJ: Describe the relationship of cardiac output, preload, afterload, contractility, and heart rate to the process of oxygenation.
TOP: Planning MSC: Physiological Adaptation
13. A patient has been diagnosed with heart failure and cardiac output is decreased. Which formula can the nurse use to calculate
cardiac output?
a. Myocardial contractility × myocardial blood flow
b. Ventricular filling time/diastolic filling time
c. Stroke volume × heart rate
d. Preload/afterload
ANS: C
Cardiac output can be calculated by multiplying the stroke volume and the heart rate. The other options are not measures of cardiac
output.
DIF: Understand (comprehension)
OBJ: Differentiate among the physiological processes of cardiac output, myocardial blood flow, and coronary artery circulation.
TOP: Assessment MSC: Physiological Adaptation
Copyright © 2021, Elsevier Inc. All rights reserved. 4
14. A patient’s heart rate increased from 94 to 164 beats/min. What will the nurse expect as a result?
a. Increase in diastolic filling time
b. Decrease in hemoglobin level
c. Decrease in cardiac output
d. Increase in stroke volume
ANS: C
With a sustained heart rate greater than 160 beats/min, diastolic filling time decreases, decreasing stroke volume, and cardiac
output. The hemoglobin level would not be affected.
DIF: Understand (comprehension)
OBJ: Describe the relationship of cardiac output, preload, afterload, contractility, and heart rate to the process of oxygenation.
TOP: Planning MSC: Physiological Adaptation
15. Which determination is the nurse trying to achieve by monitoring a patient’s cardiac output?
a. Peripheral extremity circulation
b. Oxygenation requirements
c. Presence of cardiac dysrhythmias
d. Ventilation status
ANS: A
Cardiac output indicates how much blood is being circulated systemically throughout the body to the periphery. The amount of
blood ejected from the left ventricle each minute is the cardiac output. Oxygen status would be determined by pulse oximetry and
the presence of cyanosis. Cardiac dysrhythmias are an electrical impulse monitored through ECG results. Ventilation status is
measured by respiratory rate, pulse oximetry, and capnography. Capnography provides instant information about the patient’s
ventilation. Ventilation status does not depend solely on cardiac output.
DIF: Understand (comprehension)
OBJ: Describe the relationship of cardiac output, preload, afterload, contractility, and heart rate to the process of oxygenation.
TOP: Planning MSC: Physiological Adaptation
16. A nurse is caring for a group of patients. Which patient should the nurse see first?
a. A patient with hypercapnia wearing an oxygen mask
b. A patient with a chest tube ambulating with the chest tube unclamped
c. A patient with thick secretions being tracheal suctioned first and then orally
d. A patient with a new tracheostomy and tracheostomy obturator at bedside
ANS: A
The mask is contraindicated for patients with carbon dioxide retention (hypercapnia) because retention can be worsened; the nurse
must see this patient first to correct the problem. All the rest are using correct procedures and do not need to be seen first. A chest
tube should not be clamped when ambulating. Clamping a chest tube is contraindicated when ambulating or transporting a patient.
Clamping can result in a tension pneumothorax. Use nasotracheal suctioning before pharyngeal suctioning whenever possible. The
mouth and pharynx contain more bacteria than the trachea. Keep tracheostomy obturator at bedside with a fresh (new)
tracheostomy to facilitate reinsertion of the outer cannula if dislodged.
DIF: Analyze (analysis)
OBJ: Assess for the risk factors affecting a patient’s oxygenation.
TOP: Assessment MSC: Management of Care
17. A patient has inadequate stroke volume related to decreased preload. Which treatment does the nurse prepare to administer?
a. Diuretics
b. Vasodilators
c. Chest physiotherapy
d. Intravenous (IV) fluids
ANS: D
Preload is affected by the circulating volume; if the patient has decreased fluid volume, it will need to be replaced with fluid or
blood therapy. Preload is the amount of blood in the left ventricle at the end of diastole, often referred to as end-diastolic volume.
Giving diuretics and vasodilators will make the situation worse. Diuretics cause fluid loss; the patient is already low on fluids or the
preload would not be decreased. Vasodilators reduced blood return to the heart, making the situation worse; the patient does not
have enough blood and fluid to the heart or the preload would not be decreased. Chest physiotherapy is a group of therapies for
mobilizing pulmonary secretions. Chest physiotherapy will not help this cardiovascular problem.
DIF: Apply (application)
OBJ: Describe the relationship of cardiac output, preload, afterload, contractility, and heart rate to the process of oxygenation.
TOP: Planning MSC: Management of Care
18. A nurse is preparing to suction a patient. The pulse is 65 and pulse oximetry is 94%. Which finding will cause the nurse to stop
suctioning?
a. Pulse 75
b. Pulse 80
c. Oxygen saturation 91%
d. Oxygen saturation 88%
ANS: D
Stop when oxygen saturation is 88%. Monitor patient’s vital signs and oxygen saturation during procedure; note whether there is a
change of 20 beats/min (either increase or decrease) or if pulse oximetry falls below 90% or 5% from baseline. If this occurs, stop
suctioning. A pulse rate of 75 is only 10 beats different from the start of the procedure. A pulse rate of 80 is 15 beats different from
the start of suctioning. Oxygen saturation of 91% is not 5% from baseline or below 90%.
DIF: Apply (application)
OBJ: Identify the clinical outcomes occurring as a result of hyperventilation, hypoventilation, and hypoxemia. TOP: Assessment
MSC: Reduction of Risk Potential
Copyright © 2021, Elsevier Inc. All rights reserved. 5
19. The patient is experiencing right-sided heart failure. Which finding will the nurse expect when performing an assessment?
a. Peripheral edema
b. Basilar crackles
c. Chest pain
d. Cyanosis
ANS: A
Right-sided heart failure results from inability of the right side of the heart to pump effectively, leading to a systemic backup.
Peripheral edema, distended neck veins, and weight gain are signs of right-sided failure. Basilar crackles can indicate pulmonary
congestion from left-sided heart failure. Cyanosis and chest pain result from inadequate tissue perfusion.
DIF: Apply (application)
OBJ: Assess for the physical manifestations that occur with alterations in oxygenation.
TOP: Assessment MSC: Physiological Adaptation
20. A nurse is reviewing the electrocardiogram (ECG) results. Which portion of the conduction system does the nurse consider when
evaluating the P wave?
a. SA node
b. AV node
c. Bundle of His
d. Purkinje fibers
ANS: A
The P wave represents the electrical conduction through both atria; the SA node initiates electrical conduction through the atria.
The AV node conducts down through the bundle of His and the Purkinje fibers to cause ventricular contraction.
DIF: Understand (comprehension)
OBJ: Describe the relationship of cardiac output, preload, afterload, contractility, and heart rate to the process of oxygenation.
TOP: Evaluation MSC: Physiological Adaptation
21. A nurse teaches a patient about atelectasis. Which statement by the patient indicates an understanding of atelectasis?
a. “Atelectasis affects only those with chronic conditions such as emphysema.”
b. “It is important to do breathing exercises every hour to prevent atelectasis.”
c. “If I develop atelectasis, I will need a chest tube to drain excess fluid.”
d. “Hyperventilation will open up my alveoli, preventing atelectasis.”
ANS: B
Atelectasis develops when alveoli do not expand. Breathing exercises, especially deep breathing and incentive spirometry, increase
lung volume and open the airways, preventing atelectasis. Deep breathing also opens the pores of Kohn between alveoli to allow
sharing of oxygen between alveoli. Atelectasis can affect anyone who does not deep breathe. A chest tube is for pneumothorax or
hemothorax. It is deep breathing, not hyperventilation, that prevents atelectasis.
DIF: Apply (application)
OBJ: Identify the clinical outcomes occurring as a result of hyperventilation, hypoventilation, and hypoxemia. TOP: Evaluation
MSC: Physiological Adaptation
22. The nurse is caring for a patient with respiratory problems. Which assessment finding indicates a late sign of hypoxia?
a. Elevated blood pressure
b. Increased pulse rate
c. Restlessness
d. Cyanosis
ANS: D
Cyanosis, blue discoloration of the skin and mucous membranes caused by the presence of desaturated hemoglobin in capillaries, is
a late sign of hypoxia. Elevated blood pressure, increased pulse rate, and restlessness are early signs of hypoxia.
DIF: Understand (comprehension)
OBJ: Assess for the physical manifestations that occur with alterations in oxygenation.
TOP: Assessment MSC: Physiological Adaptation
23. A nurse is caring for a 5-year-old patient whose temperature is 101.2° F. The nurse expects this patient to hyperventilate. Which
factor does the nurse remember when planning care for this type of hyperventilation?
a. Anxiety over illness
b. Decreased drive to breathe
c. Increased metabolic demands
d. Infection destroying lung tissues
ANS: C
Increased body temperature (fever) increases the metabolic rate, thereby increasing carbon dioxide production. The increased
carbon dioxide level stimulates an increase in the patient’s rate and depth of respiration, causing hyperventilation. Anxiety can
cause hyperventilation, but this is not the direct cause from a fever. Sleep causes a decreased respiratory drive; hyperventilation
speeds up breathing. The cause of the fever in this question is unknown.
DIF: Understand (comprehension)
OBJ: Discuss the effects of a patient’s level of health, age, lifestyle, and environment on oxygenation. TOP: Planning MSC:
Physiological Adaptation
Copyright © 2021, Elsevier Inc. All rights reserved. 6
24. A nurse is preparing a patient for nasotracheal suctioning. In which order will the nurse perform the steps, beginning with the first
step?
1. Insert catheter.
2. Apply suction and remove.
3. Have patient deep breathe.
4. Encourage patient to cough.
5. Attach catheter to suction system.
6. Rinse catheter and connecting tubing.
a. 1, 2, 3, 4, 5, 6
b. 4, 5, 1, 2, 3, 6
c. 5, 3, 1, 2, 4, 6
d. 3, 1, 2, 5, 4, 6
ANS: C
The steps for nasotracheal suctioning are as follows: verify that catheter is attached to suction; have patient deep breathe; insert
catheter; apply intermittent suction for no more than 10 seconds and remove; encourage patient to cough; and rinse catheter and
connecting tubing with normal saline.
DIF: Understand (comprehension)
OBJ: Describe nursing care interventions used to promote oxygenation in the primary care, acute care, and restorative and continuing care
settings. TOP: Implementation
MSC: Basic Care and Comfort
25. A patient is experiencing carbon dioxide retention from lung problems. Which type of diet will the nurse most likely suggest for
this patient?
a. Moderate-carbohydrate
b. Low-caffeine
c. High-caffeine
d. High-carbohydrate
ANS: A
A moderate-carbohydrate diet is best. Diets high in carbohydrates play a role in increasing the carbon dioxide load for patients with
carbon dioxide retention. As carbohydrates are metabolized, an increased load of carbon dioxide is created and excreted via the
lungs. A low- or high-caffeine diet is not as important as the carbohydrate load.
DIF: Apply (application)
OBJ: Discuss the effects of a patient’s level of health, age, lifestyle, and environment on oxygenation. TOP: Implementation
MSC: Basic Care and Comfort
26. A nurse caring for a patient prescribed warfarin discovers that the patient is taking garlic to help with hypertension. Which
condition will the nurse assess for in this patient?
a. Increased cholesterol level
b. Distended jugular vein
c. Bleeding
d. Angina
ANS: C
Patients taking warfarin for anticoagulation prolong the prothrombin time (PT)/international normalized ratio (INR) results if they
are taking Gingko biloba, garlic, or ginseng with the anticoagulant. The drug interaction can precipitate a life-threatening bleed.
Increased cholesterol levels are associated with saturated fat dietary intake. A distended jugular vein and peripheral edema are
associated with damage to the right side of the heart. Angina is temporary ischemia of the heart muscle.
DIF: Apply (application)
OBJ: Discuss the effects of a patient’s level of health, age, lifestyle, and environment on oxygenation. TOP: Assessment MSC:
Pharmacological and Parenteral Therapies
27. A nurse is caring for a patient whose tissue perfusion is poor as the result of hypertension. When the patient asks what to eat for
breakfast, which meal should the nurse suggest?
a. A cup of nonfat yogurt with granola and a handful of dried apricots
b. Whole wheat toast with butter and a side of bacon
c. A bowl of cereal with whole milk and a banana
d. Omelet with sausage, cheese, and onions
ANS: A
A 2000-calorie diet of fruits, vegetables, and low-fat dairy foods that are high in fiber, potassium, calcium, and magnesium and low
in saturated and total fat helps prevent and reduce the effects of hypertension. Nonfat yogurt with granola is a good source of
calcium, fiber, and potassium; dried apricots add a second source of potassium. Although cereal and a banana provide fiber and
potassium, skim milk should be substituted for whole milk to decrease fat. An omelet with sausage and cheese is high in fat. Butter
and bacon are high in fat.
DIF: Analyze (analysis)
OBJ: Develop a plan of care for a patient with altered need for oxygenation.
TOP: Planning MSC: Health Promotion and Maintenance
Copyright © 2021, Elsevier Inc. All rights reserved. 7
28. Upon auscultation of the patient’s chest, the nurse hears a whooshing sound at the fifth intercostal space. What does this finding
indicate to the nurse?
a. The beginning of the systolic phase
b. Regurgitation of the mitral valve
c. The opening of the aortic valve
d. Presence of orthopnea
ANS: B
When regurgitation occurs, there is a backflow of blood into an adjacent chamber. For example, in mitral regurgitation, the mitral
leaflets do not close completely. When the ventricles contract, blood escapes back into the atria, causing a murmur, or “whooshing”
sound. The systolic phase begins with ventricular filling and closing of the aortic valve, which is heard as the first heart sound, S1.
Orthopnea is an abnormal condition in which a patient uses multiple pillows when reclining to breathe easier or sits leaning
forward with arms elevated.
DIF: Understand (comprehension)
OBJ: Identify the clinical outcomes occurring as a result of disturbances in conduction, altered cardiac output, impaired valvular function,
myocardial ischemia, and impaired tissue perfusion.
TOP: Assessment MSC: Physiological Adaptation
29. A nurse is caring for a patient diagnosed with chronic obstructive pulmonary disease (COPD) who is receiving 2 L/min of oxygen.
Which oxygen delivery device is most appropriate for the nurse to administer the oxygen?
a. Nasal cannula
b. Simple face mask
c. Non-rebreather mask
d. Partial non-rebreather mask
ANS: A
Nasal cannulas deliver oxygen from 1 to 6 L/min. All other devices (simple face mask, non-rebreather mask, and partial
non-rebreather mask) are intended for flow rates greater than 6 L/min.
DIF: Apply (application)
OBJ: Develop a plan of care for a patient with altered need for oxygenation.
TOP: Planning MSC: Reduction of Risk Potential
30. The nurse plans to closely monitor the oxygen status of an older-adult patient undergoing anesthesia because of which age-related
change?
a. Thinner heart valves cause lipid accumulation and fibrosis.
b. Diminished respiratory muscle strength may cause poor chest expansion.
c. Alterations in mental status prevent patients’ awareness of ineffective breathing.
d. An increased number of pacemaker cells make proper anesthesia induction more
difficult.
ANS: B
Age-related changes in the thorax that occur from ossification of costal cartilage, decreased space between vertebrae, and
diminished respiratory muscle strength lead to problems with chest expansion and oxygenation, whereby the patient will have
difficulty excreting anesthesia gas. The nurse needs to monitor the patient’s oxygen status carefully to make sure the patient does
not retain too much of the drug. Older adults experience alterations in cardiac function as a result of calcification of the conduction
pathways, thicker and stiffer heart valves caused by lipid accumulation and fibrosis, and a decrease in the number of pacemaker
cells in the SA node. Altered mental status is not a normal age-related change; it indicates possible cardiac and/or respiratory
problems.
DIF: Understand (comprehension)
OBJ: Discuss the effects of a patient’s level of health, age, lifestyle, and environment on oxygenation. TOP: Assessment MSC:
Health Promotion and Maintenance
31. The nurse determines that an older-adult patient is at risk for infection due to decreased immunity. Which plan of care best
addresses the prevention of infection for the patient?
a. Inform the patient of the importance of finishing the entire dose of antibiotics.
b. Encourage the patient to stay up-to-date on all vaccinations.
c. Schedule patient to get annual tuberculosis skin testing.
d. Create an exercise routine to run 45 minutes every day.
ANS: B
A nursing care plan for preventative health measures should be reasonable and feasible. Keeping up-to-date on vaccinations is
important because vaccine reduces the severity of illnesses and serious complications. Determine if and when the patient has had a
pneumococcal or influenza (flu) vaccine. This is especially important when assessing older adults because of their increased risk
for respiratory disease. Although it is important to finish the full course of antibiotics, it is not a preventative health measure.
Scheduling annual tuberculosis skin tests does not address prevention and is an unreliable indictor of tuberculosis in older patients.
The exercise routine should be reasonable to increase compliance; exercise is recommended only 3 to 4 times a week for 30 to 60
minutes, and walking, rather than running, is an efficient method.
DIF: Apply (application)
OBJ: Describe nursing care interventions used to promote oxygenation in the primary care, acute care, and restorative and continuing care
settings. TOP: Planning
MSC: Health Promotion and Maintenance
Copyright © 2021, Elsevier Inc. All rights reserved. 8
32. The nurse is caring for a patient experiencing fluid volume overload. Which physiological effect does the nurse most likely expect?
a. Increased preload
b. Increased heart rate
c. Decreased afterload
d. Decreased tissue perfusion
ANS: A
Preload refers to the amount of blood in the left ventricle at the end of diastole; an increase in circulating volume would increase
the preload of the heart. Afterload refers to resistance; increased pressure would lead to increased resistance, and afterload would
increase. A decrease in tissue perfusion would be seen with hypovolemia. A decrease in fluid volume would cause an increase in
heart rate as the body is attempting to increase cardiac output.
DIF: Understand (comprehension)
OBJ: Describe the relationship of cardiac output, preload, afterload, contractility, and heart rate to the process of oxygenation.
TOP: Planning MSC: Physiological Adaptation
33. A nurse is caring for a patient prescribed continuous cardiac monitoring for heart dysrhythmias. Which rhythm will cause the nurse
to intervene immediately?
a. Ventricular tachycardia
b. Atrial fibrillation
c. Sinus rhythm
d. Paroxysmal supraventricular tachycardia
ANS: A
Ventricular tachycardia and ventricular fibrillation are life-threatening rhythms that require immediate intervention. Ventricular
tachycardia is a life-threatening dysrhythmia because of the decreased cardiac output and the potential to deteriorate into
ventricular fibrillation or sudden cardiac death. Atrial fibrillation is a common dysrhythmia in older adults and is not as serious as
ventricular tachycardia. Sinus rhythm is normal. Paroxysmal supraventricular tachycardia is a sudden, rapid onset of tachycardia
originating above the AV node. It often begins and ends spontaneously.
DIF: Apply (application)
OBJ: Identify the clinical outcomes occurring as a result of disturbances in conduction, altered cardiac output, impaired valvular function,
myocardial ischemia, and impaired tissue perfusion.
TOP: Implementation MSC: Management of Care
34. The patient is experiencing angina pectoris. Which assessment finding does the nurse expect when conducting a history and
physical examination?
a. Experiences chest pain after eating a heavy meal.
b. Experiences adequate oxygen saturation during exercise.
c. Experiences crushing chest pain for more than 20 minutes.
d. Experiences tingling in the left arm that lasts throughout the morning.
ANS: A
Angina pectoris is chest pain that results from limited oxygen supply. Often pain is precipitated by activities such as exercise,
stress, and eating a heavy meal and lasts 3 to 5 minutes. Symptoms of angina pectoris are relieved by rest and/or nitroglycerin.
Adequate oxygen saturation occurs with rest; inadequate oxygen saturation occurs during exercise. Pain lasting longer than 20
minutes or arm tingling that persists could be a sign of myocardial infarction.
DIF: Apply (application)
OBJ: Assess for the physical manifestations that occur with alterations in oxygenation.
TOP: Assessment MSC: Physiological Adaptation
35. Which risk factor for cardiopulmonary disease should the nurse describe as modifiable?
a. Stress
b. Allergies
c. Family history
d. Gender
ANS: A
Young and middle-age adults are exposed to multiple cardiopulmonary risk factors: an unhealthy diet, lack of exercise, stress,
over-the-counter and prescription drugs not used as intended, illegal substances, and smoking. Reducing these modifiable factors
decreases a patient’s risk for cardiac or pulmonary diseases. A nonmodifiable risk factor is family history; determine familial risk
factors such as a family history of lung cancer or cardiovascular disease. Other nonmodifiable risk factors include allergies and
gender.
DIF: Understand (comprehension)
OBJ: Assess for the risk factors affecting a patient’s oxygenation.
TOP: Assessment MSC: Health Promotion and Maintenance
Copyright © 2021, Elsevier Inc. All rights reserved. 9
36. The nurse is creating a plan of care for an obese patient who is experiencing fatigue related to ineffective breathing. Which
intervention best addresses a short-term goal the patient could achieve?
a. Sleeping on two to three pillows at night
b. Sensibly reducing daily calorie intake
c. Running 30 minutes every morning
d. Stopping smoking immediately
ANS: A
To achieve a short-term goal, the nurse should plan a lifestyle change that the patient can make immediately that will have a quick
effect. Sleeping on several pillows at night will immediately relieve orthopnea and open the patient’s airway, thereby reducing
fatigue. Running 30 minutes a day will improve cardiopulmonary health, but a patient needs to build up exercise tolerance.
Smoking cessation is another process that many people have difficulty doing immediately. A more realistic short-term goal would
be to gradually reduce the number of cigarettes smoked. Limiting caloric intake can help a patient lose weight, but this is a gradual
process and is not reasonable for a short-term goal.
DIF: Apply (application)
OBJ: Develop a plan of care for a patient with altered need for oxygenation.
TOP: Planning MSC: Basic Care and Comfort
37. A patient experiencing left-sided hemiparesis has developed bronchitis and has a heart rate of 105 beats/min, blood pressure of
156/90 mm Hg, and respiration rate of 30 breaths/min. Which nursing diagnosis is a priority?
a. Risk for skin breakdown
b. Impaired gas exchange
c. Activity intolerance
d. Risk for infection
ANS: B
The most important nursing intervention is to maintain airway and circulation for this patient; therefore, Impaired gas exchange is
the first nursing priority. Activity intolerance is a concern but is not the priority in this case. Risk for skin breakdown and Risk for
infection are also important but do not address an immediate impairment with physiological integrity.
DIF: Analyze (analysis)
OBJ: Develop a plan of care for a patient with altered need for oxygenation.
TOP: Diagnosis MSC: Management of Care
38. Which nursing intervention is most effective in preventing hospital-acquired pneumonia in an older-adult patient?
a. Discontinue the humidification delivery device to keep excess fluid from lungs.
b. Monitor oxygen saturation, and frequently auscultate lung bases.
c. Assist the patient to cough, turn, and deep breathe every 2 hours.
d. Decrease fluid intake to 300 mL a shift.
ANS: C
The goal of the nursing action should be the prevention of pneumonia; the action that best addresses this is to cough, turn, and deep
breathe to keep secretions from pooling at the base of the lungs. Humidification thins respiratory secretions, making them easier to
expel and should be used. Monitoring oxygen status is important but is not a method of prevention. Hydration assists in preventing
hospital-acquired pneumonia. The best way to maintain thin secretions is to provide a fluid intake of 1500 to 2500 mL/day unless
contraindicated by cardiac or renal status. Restricting fluids is contraindicated in this situation since there is no data indicating
cardiac or renal disease.
DIF: Apply (application)
OBJ: Describe nursing care interventions used to promote oxygenation in the primary care, acute care, and restorative and continuing care
settings. TOP: Implementation
MSC: Reduction of Risk Potential
39. The nurse is assessing a patient diagnosed with emphysema. Which assessment finding requires further follow-up with the health
care provider?
a. Increased anterior-posterior diameter of the chest
b. Accessory muscle used for breathing
c. Clubbing of the fingers
d. Hemoptysis
ANS: D
Hemoptysis is an abnormal occurrence of emphysema, and further diagnostic studies are needed to determine the cause of blood in
the sputum. Clubbing of the fingers, barrel chest (increased anterior-posterior chest diameter), and accessory muscle used are all
normal findings in a patient with emphysema.
DIF: Apply (application)
OBJ: Assess for the physical manifestations that occur with alterations in oxygenation.
TOP: Assessment MSC: Management of Care
Copyright © 2021, Elsevier Inc. All rights reserved. 10
40. A patient diagnosed with chronic obstructive pulmonary disease (COPD) asks the nurse why clubbing occurs. Which response by
the nurse is most therapeutic?
a. “Your disease doesn’t send enough oxygen to your fingers.”
b. “Your disease affects both your lungs and your heart, and not enough blood is
being pumped.”
c. “Your disease will be helped if you pursed-lip breathe.”
d. “Your disease often makes patients lose mental status.”
ANS: A
Clubbing of the nail bed can occur with COPD and other diseases that cause prolonged oxygen deficiency or chronic hypoxemia.
Pursed-lipped breathing helps the alveoli stay open but is not the cause of clubbing. Loss of mental status is not a normal finding
with COPD and will not result in clubbing. Low oxygen and not low circulating blood volume is the problem in COPD that results
in clubbing.
DIF: Apply (application)
OBJ: Identify the clinical outcomes occurring as a result of hyperventilation, hypoventilation, and hypoxemia. TOP: Teaching/Learning
MSC: Physiological Adaptation
41. A patient experiencing a pneumothorax has a chest tube inserted and is placed on low constant suction. Which finding requires
immediate action by the nurse?
a. The patient reports pain at the chest tube insertion site that increases with
movement.
b. Fifty milliliters of blood gushes into the drainage device after the patient coughs.
c. No bubbling is present in the suction control chamber of the drainage device.
d. Yellow purulent discharge is seen leaking out from around the dressing site.
ANS: C
No bubbling in the suction control chamber indicates an obstruction of the drainage system. An obstruction causes increased
pressure, which can cause a tension pneumothorax, which can be life threatening. The nurse needs to determine whether the leak is
inside the thorax or in the tubing and act from there. Occasional blood gushes from the lung owing to lung expansion, as during a
cough; this is reserve drainage. Drainage over 100 mL/hr after 3 hours of chest tube placement is cause for concern. Yellow
purulent drainage indicates an infection that should be reported to the health care provider but is not as immediately life threatening
as the lack of bubbling in the suction control chamber.
DIF: Apply (application)
OBJ: Describe nursing care interventions used to promote oxygenation in the primary care, acute care, and restorative and continuing care
settings. TOP: Implementation
MSC: Management of Care
42. The nurse is caring for a patient who has had a tracheostomy tube inserted. Which nursing intervention is most effective in
promoting effective airway clearance?
a. Suctioning respiratory secretions several times every hour
b. Administering humidified oxygen through a tracheostomy collar
c. Instilling normal saline into the tracheostomy to thin secretions before suctioning
d. Deflating the tracheostomy cuff before allowing the patient to cough up secretions
ANS: B
Humidification from air humidifiers or humidified oxygen tracheostomy collars can help prevent drying of secretions that cause
occlusion. Suctioning should be done only as needed; too frequent suctioning can damage the mucosal lining, resulting in thicker
secretions. Normal saline should not be instilled into a tracheostomy; research showed no benefit with this technique. The purpose
of the tracheostomy cuff is to keep secretions from entering the lungs; the nurse should not deflate the tracheostomy cuff unless
instructed to do so by the health care provider.
DIF: Apply (application)
OBJ: Develop a plan of care for a patient with altered need for oxygenation.
TOP: Implementation MSC: Physiological Adaptation
43. The nurse is educating a student nurse on caring for a patient with a chest tube. Which statement from the student nurse indicates
successful learning?
a. “I should clamp the chest tube when giving the patient a bed bath.”
b. “I should report if I see continuous bubbling in the water-seal chamber.”
c. “I should strip the drains on the chest tube every hour to promote drainage.”
d. “I should notify the health care provider first, if the chest tube becomes
dislodged.”
ANS: B
Correct care of a chest tube involves knowing normal and abnormal functioning of the tube. A constant or intermittent bubbling in
the water-seal chamber indicates a leak in the drainage system, and the health care provider must be notified immediately. Stripping
the tube is not routinely performed as it increases pressure. If the tubing disconnects from the drainage unit, instruct the patient to
exhale as much as possible and to cough. This maneuver rids the pleural space of as much air as possible. Temporarily reestablish a
water seal by immersing the open end of the chest tube into a container of sterile water. The chest tube should not be clamped
unless necessary; if so, the length of time clamped would be minimal to reduce the risk of pneumothorax.
DIF: Apply (application)
OBJ: Describe nursing care interventions used to promote oxygenation in the primary care, acute care, and restorative and continuing care
settings. TOP: Evaluation
MSC: Management of Care
Copyright © 2021, Elsevier Inc. All rights reserved. 11
44. Which coughing technique will the nurse use to help a patient clear central airways?
a. Huff
b. Quad
c. Cascade
d. Incentive spirometry
ANS: A
The huff cough stimulates a natural cough reflex and is generally effective only for clearing central airways. While exhaling, the
patient opens the glottis by saying the word huff. The quad cough technique is for patients without abdominal muscle control such
as those with spinal cord injuries. While the patient breathes out with a maximal expiratory effort, the patient or nurse pushes
inward and upward on the abdominal muscles toward the diaphragm, causing the cough. With the cascade cough the patient takes a
slow, deep breath and holds it for 2 seconds while contracting expiratory muscles. Then he or she opens the mouth and performs a
series of coughs throughout exhalation, thereby coughing at progressively lowered lung volumes. This technique promotes airway
clearance and a patent airway in patients with large volumes of sputum. Incentive spirometry encourages voluntary deep breathing
by providing visual feedback to patients about inspiratory volume. It promotes deep breathing and prevents or treats atelectasis in
the postoperative patient.
DIF: Understand (comprehension)
OBJ: Describe nursing care interventions used to promote oxygenation in the primary care, acute care, and restorative and continuing care
settings. TOP: Implementation
MSC: Physiological Adaptation
45. The nurse is suctioning a patient with a tracheostomy tube. Which action will the nurse take?
a. Set suction regulator at 150 to 200 mm Hg.
b. Limit the length of suctioning to 10 seconds.
c. Apply suction while gently rotating and inserting the catheter.
d. Liberally lubricate the end of the suction catheter with a water-soluble solution.
ANS: B
Suctioning passes should be limited to 10 seconds to avoid hypoxemia. Suction for a tracheostomy should be set at 100 to 150 mm
Hg. Excessive lubrication can clog the catheter or occlude the airway; lubricant is not necessary for oropharyngeal or artificial
airway (tracheostomy) suctioning. Suction should never be applied on insertion.
DIF: Apply (application)
OBJ: Describe nursing care interventions used to promote oxygenation in the primary care, acute care, and restorative and continuing care
settings. TOP: Implementation
MSC: Reduction of Risk Potential
46. The nurse is caring for a patient who is prescribed oxygen via a nasal cannula. Which task can the nurse delegate to the nursing
assistive personnel?
a. Applying the nasal cannula
b. Adjusting the oxygen flow
c. Assessing lung sounds
d. Setting up the oxygen
ANS: A
The skill of applying (not adjusting oxygen flow) a nasal cannula or oxygen mask can be delegated to assistive personnel (AP). The
nurse is responsible for assessing the patient’s respiratory system, response to oxygen therapy, and setup of oxygen therapy,
including adjustment of oxygen flow rate.
DIF: Apply (application)
OBJ: Describe nursing care interventions used to promote oxygenation in the primary care, acute care, and restorative and continuing care
settings. TOP: Planning
MSC: Management of Care
47. The nurse is using a closed suction device. Which patient will be most appropriate for this suctioning method?
a. A 5 year old with excessive drooling from epiglottitis
b. A 5 year old with an asthma attack following severe allergies
c. A 24 year old with a right pneumothorax following a motor vehicle accident
d. A 24 year old with acute respiratory distress syndrome requiring mechanical
ventilation
ANS: D
Closed suctioning is most often used on patients who require invasive mechanical ventilation to support their respiratory efforts
because it permits continuous delivery of oxygen while suction is performed and reduces the risk of oxygen desaturation. In this
case, the acute respiratory distress syndrome requires mechanical ventilation. In the presence of epiglottitis, croup, laryngospasm,
or irritable airway, the entrance of a suction catheter via the nasal route causes intractable coughing, hypoxemia, and severe
bronchospasm, necessitating emergency intubation or tracheostomy. The 5-year-old child with asthma would benefit from an
inhaler. A chest tube is needed for the pneumothorax.
DIF: Analyze (analysis)
OBJ: Discuss the effects of a patient’s level of health, age, lifestyle, and environment on oxygenation. TOP: Implementation
MSC: Physiological Adaptation
Copyright © 2021, Elsevier Inc. All rights reserved. 12
48. While the nurse is changing the ties on a tracheostomy collar, the patient coughs, dislodging the tracheostomy tube. Which action
will the nurse take first?
a. Press the emergency response button.
b. Insert a spare tracheostomy with the obturator.
c. Manually occlude the tracheostomy with sterile gauze.
d. Place a face mask delivering 100% oxygen over the nose and mouth.
ANS: B
The nurse’s first priority is to establish a stable airway by inserting a spare trach into the patient’s airway; ideally an obturator
should be used. The nurse could activate the emergency response team if the patient is still unstable after the tracheostomy is
placed. A patient with a tracheostomy breathes through the tube, not the nose or mouth; a face mask would not be an effective
method of getting air into the lungs. Manually occluding pressure over the tracheostomy site is not appropriate and would block the
patient’s only airway.
DIF: Apply (application)
OBJ: Develop a plan of care for a patient with altered need for oxygenation.
TOP: Implementation MSC: Management of Care
MULTIPLE RESPONSE
1. A nurse is following the How-to Guide to prevent ventilator-associated pneumonia. Which strategies is the nurse using? (Select all
that apply.)
a. Head of bed elevation to 90 degrees at all times
b. Daily oral care with chlorhexidine
c. Delirium monitoring
d. Clean technique when suctioning
e. Daily “sedation vacations”
f. Heart failure prophylaxis
ANS: B, C, E
The key components of the Institute for Healthcare Improvement (IHI) How-to Guide are:
Elevation of the head of the bed (HOB)—elevation is greater than 30 degrees
Daily “sedation vacations” and assessment of readiness to extubate
Peptic ulcer disease prophylaxis
Venous thromboembolism prophylaxis
Daily oral care with chlorhexidine
Delirium monitoring
Early ambulation
Sterile technique is used for suctioning when on ventilators. Heart failure prophylaxis is not a component.
DIF: Apply (application)
OBJ: Describe nursing care interventions used to promote oxygenation in the primary care, acute care, and restorative and continuing care
settings. TOP: Implementation
MSC: Reduction of Risk Potential
2. A nurse is teaching a community health promotion class and discusses the flu vaccine. Which information will the nurse include in
the teaching session? (Select all that apply.)
a. It is given yearly.
b. It is given in a series of four doses.
c. It is safe for children allergic to eggs.
d. It is safe for adults with acute febrile illnesses.
e. The live, attenuated nasal spray is given to people over 50.
f. The vaccines are recommended for all people 6 months and older.
ANS: A, F
Annual (yearly) flu vaccines are recommended for all people 6 months and older. People with a known hypersensitivity to eggs or
other components of the vaccine should consult their health care provider before being vaccinated. There is a flu vaccine made
without egg proteins that is approved for adults 18 years of age and older. Adults with an acute febrile illness should schedule the
vaccination after they have recovered. The live, attenuated nasal spray vaccine is given to people from 2 through 49 years of age if
they are not pregnant or do not have certain long-term health problems such as asthma; heart, lung, or kidney disease; diabetes; or
anemia.
DIF: Apply (application)
OBJ: Describe nursing care interventions used to promote oxygenation in the primary care, acute care, and restorative and continuing care
settings. TOP: Teaching/Learning
MSC: Health Promotion and Maintenance
Copyright © 2021, Elsevier Inc. All rights reserved. 13
3. A nurse is caring for a patient being treated for sleep apnea. Which types of ventilator support should the nurse be prepared to
administer for this patient? (Select all that apply.)
a. Assist-control (AC)
b. Pressure support ventilation (PSV)
c. Bilevel positive airway pressure (BiPAP)
d. Continuous positive airway pressure (CPAP)
e. Synchronized intermittent mandatory ventilation (SIMV)
ANS: C, D
Ventilatory support is achieved using a variety of modes, including continuous positive airway pressure (CPAP) and bilevel
positive airway pressure (BiPAP). The purpose of CPAP and BiPAP is to maintain a positive airway pressure and improve alveolar
ventilation. This prevents or treats atelectasis by inflating the alveoli, reducing pulmonary edema by forcing fluid out of the lungs
back into circulation, and improving oxygenation in those with sleep apnea. AC, PSV, and SIMV are invasive mechanical
ventilation and are not routinely used on patients with sleep apnea. AC delivers a set tidal volume (VT) with each breath, regardless
of whether the breath was triggered by the patient or the ventilator. Synchronized intermittent mandatory ventilation like AC
delivers a minimum number of fully assisted breaths per minute that are synchronized with the patient’s respiratory effort. Any
breaths taken between volume-cycled breaths are not assisted; the volume of these breaths is determined by the patient’s strength,
effort, and lung mechanics. PSV mode is often combined with SIMV mode: inspiratory pressure is added to spontaneous breaths to
overcome the resistance of the endotracheal tube or to help increase the volume of the patient’s spontaneous breaths.
DIF: Apply (application)
OBJ: Describe nursing care interventions used to promote oxygenation in the primary care, acute care, and restorative and continuing care
settings. TOP: Planning
MSC: Physiological Adaptation